Despite being (IMO) a philosophy blog, many Less Wrongers tend to disparage mainstream philosophy and emphasize the divergence between our beliefs and theirs. But, how different are we really? My intention with this post is to quantify this difference.

The questions I will post as comments to this article are from the 2009 PhilPapers Survey. If you answer "other" on any of the questions, then please reply to that comment in order to elaborate your answer. Later, I'll post another article comparing the answers I obtain from Less Wrongers with those given by the professional philosophers. This should give us some indication about the differences in belief between Less Wrong and mainstream philosophy.

Glossary

analytic-synthetic distinction, A-theory and B-theory, atheism, compatibilism, consequentialism, contextualism, correspondence theory of truth, deontology, egalitarianism, empiricism, Humeanism, libertarianism, mental content externalism, moral realism, moral motivation internalism and externalism, naturalism, nominalism, Newcomb's problem, physicalism, Platonism, rationalism, relativism, scientific realism, trolley problem, theism, virtue ethics

Note

Thanks pragmatist, for attaching short (mostly accurate) descriptions of the philosophical positions under the poll comments.

Post Script

The polls stopped rendering correctly after the migration to LW 2.0, but the raw data can be found in this repo.

New Comment
629 comments, sorted by Click to highlight new comments since: Today at 3:44 AM
Some comments are truncated due to high volume. (⌘F to expand all)Change truncation settings

One respect in which Less Wrongers resemble mainstream philosophers is that many mainstream philosophers disparage mainstream philosophers and emphasize the divergence between their beliefs and those of rival mainstream philosophers. Indeed, that is something of a tradition in Western philosophy.

I've posted brief explanations for some of the questions as replies to those questions. I haven't posted explanations for those questions that I believe the vast majority of LW users will understand. If you don't understand a question, I'm fairly certain that if you scroll down far enough you'll find a comment from me with an attempt at explication.

Thanks for the clarifications, without them the questions made little sense to me. (Well, with them most polls appear poorly defined false dichotomy, but at least this unfortunate fact becomes clear).

2komponisto11y
Unfortunately, I didn't on the libertarianism/egalitarianism one. (I had a plausible guess, but I wanted to be sure that guess was right.)
2Document11y
While that improves the situation, we're still trusting that the PhilPapers respondents' beliefs about the terms perfectly match the definitions you posted. Too bad we can't survey both groups ourselves (or something).

Beware that some words might mean different things to different communities. For example, if a philosopher calls himself/herself an "anti-reductive naturalist," there's a good chance they are a strict reductionist in the LW sense. It may help to read the "thoughts on specific questions" section of this page of the PhilPapers Survey site.

7Jayson_Virissimo11y
Excellent point. I'll add a glossary to the article sometime within the next 24 hours in order to diminish some of the confusion.

I've tried to do that already, adding comments below each question that I think might be confusing.

3Jayson_Virissimo11y
Thanks for that.
[-][anonymous]11y190

Stop saying these questions are false dichotomies! None of them are, because they all have an 'other' option!

It would be interesting to have "how well do you think you understand the question?" parallel to each question. I'd imagine less consistency on questions where most participants had to look up the terms on Wikipedia prior to answering.

3Jayson_Virissimo11y
I won't object to people attaching polls to my poll comments, but I won't make a precommitment to making use of them in my analysis of the results.

Normative ethics: consequentialism, deontology or virtue ethics?

[pollid:86]

Consequentialism: The morality of actions depends only on their consequences.

Deontology: There are moral principles that forbid certain actions and encourage other actions purely based on the nature of the action itself, not on its consequences.

Virtue ethics: Ethical theory should not be in the business of evaluating actions. It should be in the business of evaluating character traits. The fundamental question of ethics is not "What makes an action right or wrong?" It is "What makes a person good or bad?"

All three in weighted combination, with consequentialism scaling such that it becomes dominant in high-stakes scenarios but is not dominant elsewhere. I believe that consequentialism, deontology and virtue ethics are mutually reducible and mutually justifying, but that flattening them into any one of the three is bad because it raises the error rate, by making some values much harder to describe and eliminating redundancy in values that would have protected them from corruption.

Thinking about this...

So, yes, in many cases I make decisions based on moral principles, because the alternatives are computationally intractable. And in a few cases I judge character traits as a proxy for doing either. And I endorse all of that, under the circumstances. Which sounds like what you're describing.

But if I discovered that one of my moral principles was causing me to act in ways that had consequences I anti-value, I would endorse discarding that principle. Which seems to me like I'm a consequentialist who sometimes uses moral principles as a processing shortcut.

Were I actually a deontologist, as described here, presumably I would shrug my shoulders, perhaps regret the negative consequences of my moral principle (perhaps not), and go on using it.

Admittedly, I'm not sure I have a crisp understanding of the distinction between moral principles (which consequentialism on this account ignores) and values (on which it depends).

0Matt_Simpson11y
I voted "other" for the same reason, though I'm less certain about virtue ethics being being equivalent to the other two.

I lean toward Consequentialism but I support something like deontology/virtue ethics for reasons of personal computability.

8drnickbone11y
How should we vote for "rule consequentialism"? I went for "Lean toward consequentialism" though it is arguably a form of deontology. "Other" is not very precise.
2thomblake11y
Rule consequentialism is either consequentialism or deontology (or just inconsistent). What makes it the case that you should follow the rules? If it is that the following the rules maximizes expected utility, then it's ultimately consequentialism. Otherwise, it's most likely deontology.
0drnickbone11y
A common formulation is that the "rules" are the ones which if generally adopted as a moral code would maximize expected utility: i.e. there is a form of "best" or "ideal" moral code. However, this can lead to cases where an act which would (by itself) maximize expected utility would also be in violation of the ideal moral code. So the act would be "right" from an act utilitarian point of view, but "wrong" from a rule utilitarian point of view. Relevant examples here could include torturing someone "for a greater good" (such as to stop the infamous ticking time bomb). The logic for torture in such cases seems very sound from an act utilitarian perspective; however, an ideal moral code would have a rule of the form "Don't torture anyone, ever, for any reason, no matter if it appears to lead to a greater good". This, incidentally, is one resolution to Torture vs Dust Specks.
0thomblake11y
Right, but if the moral code is really ideal on consequentialist grounds and following the rules really leads to better expected outcomes for humans than not doing so, even when it appears otherwise, then the act consequentialist should also agree that you should follow the rule even when it appears to be sub-optimal. On the other hand, if the claim is that an ideal reasoner with full knowledge should follow the rule even when it provably does not maximize expected utility, then that's a form of deontology and a consequentialist should disagree.
3drnickbone11y
There is a recognized distinction here between a moral decision procedure and the criterion for an action to be right or wrong. Pretty much all serious act utilitarians approve a rule-utilitarian decision procedure i.e. they recommend moral agents to follow the usual moral rules (or heuristics) even in those cases where the agent believes that departing from the rules would lead to better consequences. The justification for such a decision procedure is of course that humans are not ideal reasoners, we can not predict and evaluate all consequences of our actions (including others imitating us), we do not have an ideal, impartial conception of the good, and we tend to get things horribly wrong when we depart from the rules with the best of intentions. Yet still, from an act utilitarian criterion for "right" and "wrong" a rule-violating action which maximizes expected utility is "right". This leads to some odd situations, whereby the act utilitarian would have to (privately) classify such a rule-violating action as right, but publically condemn it, call it the "wrong choice", quite possibly punish it, and generally discourage people from following it!
0thomblake11y
Yes, I was (improperly) ignoring the typically backward-looking nature of act utilitarianism. I kept saying "maximize expected utility" rather than "maximize utility" which resulted in true statements that did not reflect what act utilitarians really say. I blame the principle of charity. EDIT: And if I were being really careful, I'd make sure to phrase "maximize expected utility" in such a way that it's clear that you're maximizing the utility according to your expectations, not maximizing your expectations of utility (wireheading).
7magfrump11y
I accept consequentialism but I also believe that "acting like I'm following virtue ethics" tends to have the best consequences.
5A1987dM11y
Voted for "lean toward consequentialism". As someone once put, I consider the “fundamental” rules to be consequentialist¹, but some of the approximations I use because the fundamental rules are infeasible to calculate from scratch every time resemble deontology or virtue ethics, much like QFT and GR are time-reversal symmetric but thermodynamics isn't. Also, ethical injunctions (i.e. fudge factors in my prior probability that certain behaviours will harm someone to compensate for cognitive biases) and TDT-like game-/decision-theoretical considerations make some of my choices resemble deontology, and a term in my utility function for how awesome I am make some of my choices resemble virtue ethics. 1. I assume that, despite the name, people here don't take consequentialism to imply strictly CDT. I still think that in the True Prisoner's Dilemma against a paperclip maximizer known to use the same decision algorithms as ourselves it's immoral to defect.
0A1987dM11y
May the reason why so many philosophers don't vote for consquentialism is that they're thinking about pure CDTical act consequentialism?
4komponisto11y
Depends again on the level of discourse. Ultimately consequentialism, but a whole lot of deontology and virtue ethics in "real life".
4pragmatist11y
Moral particularism
4lukeprog11y
For the record, I consider myself a consequentialist who is also a moral particularist.
1pragmatist11y
Fair enough. I should have been more specific. I'm a particularist who thinks consequentialist reasoning is appropriate in certain contexts, but deontological reasoning is appropriate in other contexts. So I'm pretty sure "Other" is the right pick for me.
0[anonymous]11y
Is that possible? Can you both think a) that one should in general act so as to maximise happiness/utility/whatever, and b) there are no general moral rules? I think that's a contradiction.
4pragmatist11y
Consequentialism doesn't require a commitment to maximization of any particular variable. It's the claim that only the consequences of actions are relevant to moral evaluation of the actions. I think that's a weak enough claim that you can't really call it a general moral principle. So one could believe that only consequences are morally relevant, but the way in which one evaluates actions based on their consequences does not conform to any general principle. If Luke had said that he's a utilitarian who is also a particularist, that would have been a contradiction.
2[anonymous]11y
That's a good point. So I should take from Luke's claim that he does not believe one should (as a moral rule) maximise expected utility, or anything like that? And that he would say that it's possible (if perhaps unlikely) for an action to be good even if it minimizes expected utility?
0pragmatist11y
I probably shouldn't speak for Luke, but I'm guessing the answer to this is yes. If it isn't, then I don't understand how he's a particularist. I don't see why he should be committed to this claim.
0Manfred11y
I took it to mean that Luke is requiring an agent to be at least somewhat consequentialist before he even thinks of it in terms of a morality.

I don't know the definition of any of the "-ism"s. Should I not answer the questions? I imagine that others will be in the same position as I am.

EDIT: Thanks to pragmatist for the explanations!

1Jayson_Virissimo11y
If you can spare the time, then read pragmatists excellent summaries or click on the link of the term you don't understand in my glossary. Otherwise, only answer the ones you understand.

Meta-ethics: moral realism or moral anti-realism?

[pollid:84]

[EDIT: The way I had initially described the distinction was misleading, as pointed out by thomblake. I apologize for potentially skewing the results of the poll, although I don't think my revised version is that far off from the earlier version. Still, I should have been more careful.]

Moral realism: There are objective moral facts, i.e. there are facts about what is right and wrong (or good and bad) that are not constituted by a subject's beliefs and desires.

Moral anti-realism: The denial of moral realism.

4thomblake11y
Is that right? I've understood that you can be a realist about subject-sensitive objective moral facts. Is that different from saying that the facts are "agent-relative"?
2pragmatist11y
You're right, my potted descriptions here are misleading. Certain forms of relativism are appropriately classified as realist. I'll edit my descriptions.
0thomblake11y
Thanks! I was concerned I had it wrong.
0komponisto11y
Other : depends on the level of the desires (object-level, meta-level, etc.)
6lukeprog11y
My 'Other' answer is "Depends what you mean."
2[anonymous]11y
From that article: But this is orthogonal to the question of moral realism: you can have realism just as well with or without moral universalism. So I think you're just a moral realist.
3Nisan11y
I've never seen a completely satisfying reduction of moral facts, and I don't know whether such a successful reduction would be a vindicative or an eliminative one. Am I a moral realist or an anti-realist?
0EricHerboso11y
That sounds like "undecided" to me.
3asparisi11y
Other: While there are objective "moral facts" this is because we are implying various subjective human values into the word 'moral.' Given the word 'moral' there are certain facts about what that behavior is like, but they are not "out there in the world" and are highly contextual.
2Rob Bensinger11y
Lean toward: moral realism. My leanings are semantic. There is not one unified object or semantic value that all members of our linguistic community intend by 'moral.' So we must either choose one of the semantic values in advance, or leave all moral statements underdetermined. I think choosing a semantic value that naturalizes morality (e.g., 'morality is behaving in accord with a decision procedure that optimizes for the overall preference satisfaction of all preference-bearers') is much more useful and conducive to our goals than choosing one that forfeits all moral convictions to the Dark Side. If we want to win, we have to convince people in general to move toward a LessWrongier perspective; and convincing people without making any appeal to words like 'good,' 'bad,' 'right,' 'wrong,' 'moral,' or 'immoral' is a hell of a handicap.
0[anonymous]11y
Other: There is only one right morality criteria, but it is not written into the fabric of the universe or anything. It is the idealized criteria that humans refer to as "right". It is not relative to belief or individual preferences. What I care about (my utility function) includes this abstract morality, but also very relative things like "I like human females" and "I'm more important than you". I'd say accept moral antirealism except that that is too easily construed as relativism or some other philosophy.
0Matt_Simpson11y
Other: I think I'm a realist, but it really depends on the definition of "desires."
0RobertLumley11y
For the record, I misread pragmatist's definition. My answer, which was Lean toward moral realism, should have been lean toward moral anti-realism. (I missed the "that are not agent-relative" part.)
0thomblake11y
Yeah, I'm not sure that part is correct, or it needs clarification.
0William_Quixote11y
I tend to view game strategies that lead to the best stable equilibrium as moral injunctions ( tit for tat, cooperate first) These are provable ( under certain assumptions) so I lean towards saying they are "real"
0Jayson_Virissimo11y
Yeah, I have similar ideas. On the other hand, rules that are Nash equilibria in the current "environment" are in some ways determined by the preferences of the (by now, long dead) agents (and even their initial bargaining positions). I'm having a hard time deciding how to categorize this kind of "morality" (if it can, in truth, be called such a thing). I ended up going with "Lean toward: moral anti-realism".

Meta:

IAWYC but it is slightly problematic that the Philpapers survey polls the opinions of all philosophers, rather than those in a specific field. I am unsure if the opinions on current debates in metaphysics held but political philosophers will be much better than an average college graduate's. It might be interesting to contrast the 'lesswrong position' on question X with the position of 'mainstream philosophers' who study the relevant sub-field.

You can filter the survey results by specialization. Use the AOS (Area of Specialization) drop down menu.

All I can say is that you're not going to be happy with the 'philosophy of religion' statistics.

Hopefully some of these questions will be folded into Yvain's yearly survey.

It's too late now, but if you put all the questions in the same comment then it's less work to vote in all of them and you can see correlations between answers to the different questions.

2Jayson_Virissimo11y
Yes, but then how would you handle the "Other" options?
1jimrandomh11y
They'd end up less organized, buy you could still ask people to comment; they'd just have to say which answer went with which 'Other' option.
6TheOtherDave11y
I prefer the current approach; I am not answering the questions all at once.

Science: scientific anti-realism or scientific realism?

[pollid:88]

Scientific anti-realism: While there may be strong reasons to believe in the empirical predictions of our best scientific theories, there are no strong reasons to believe in their theoretical claims about unobservable entities (such as quarks).

Scientific realism: There are strong reasons to believe in the theoretical claims about unobservable entities made by our best scientific theories.

0DanArmak11y
What is and isn't observable changes over time. Quarks are not in principle unobservable. Conversely, we couldn't observe Neptune before we had telescopes or spaceships, but surely no philosophers would argue that the scientists who predicted its existence due to gravitational influences on Uranus's orbit should have disbelieved in the theory's unobservable predictions.
5pragmatist11y
Most scientific anti-realists acknowledge this. If quarks become observable then there would be good reason to believe in them. But the mere fact that they are part of an empirically successful theory is not sufficient reason. Of course, it's unclear whether the observable/unobservable distinction makes sense. Does seeing something through a microscope count as observing it? How about an electron microscope? How about tracks in a bubble chamber?
0DanArmak11y
I hope you mean "if they are actually observed". But, if we didn't believe (to a degree) in theoretical predictions before making observations to confirm them, then we wouldn't know what observations to attempt, and would almost never actually observe something useful! Anchoring on what most humans can observe unaided is just silly. I have acute myopia since age 6; without modern glasses I wouldn't be able to observe the moon in the sky - or to read about any scientific theories. Should I discount them on that account? Or if someone were born with unusually fine eyesight, making them the only person able to observe a tiny mote of dust - should everyone else disbelieve them? If a trained dog barks when he smells explosives, which humans can't smell, should we ignore the dog?
5pragmatist11y
Yeah, that's it. The anti-realist doesn't say that we don't believe in theoretical entities. She says that we don't have strong reason to believe in them. I suspect most of us believe in things we don't have strong reason to believe in. It might be an anthropological fact that scientists tend to believe in theoretical entities and use these beliefs as guides to future research. Anti-realists don't want to deny this fact, they want to deny that the scientists' epistemic attitude (prior to making the requisite observations) is justified. I agree. This is what I was trying to say when I said the distinction between observable and unobservable doesn't make sense. It would be silly to construe "observable" as "observable without technological aids", but once one allows technological aids, where do you draw the line? We have experimental verification of quantum chromodynamics. Why doesn't this count as (extremely indirect and mediated) observation of quarks?
0RichardHughes11y
I felt this was a confused question for the reasons you've defined and so I've voted other.
3prase11y
Other: the distinction between scientific realism and anti-realism is mostly meaningless. If a scientific theory makes claims about "unobservable entities" (not sure what exactly it means), either these claims are logically entangled with testable claims that have been experimentally verified and then the reasons to believe in them are as good as the reasons to believe the verified claims (supposedly about "observable objects") or these claims are independent of the rest of the theory and presumably untestable, which means that the theory is un-Occamian and shouldn't be considered good scientific theory.
2[anonymous]11y
I think that's just realism.
1Protagoras11y
I think you've given a good example of why this question is so problematic, because what prase said sounds more like anti-realism to me. Though mostly my view is that the more you try to figure out what "realism" could mean, the less sure you will be about it; I picked "lean anti-realism" myself because I think "realism" is shorthand for a huge bundle of claims that really should be unbundled and evaluated separately.
0[anonymous]11y
As befits your handle! I took Prase to be a realist because he said that when a good scientific theory posits an unobservable entity which has such and such observable effects on observable entities, we have as much reason to believe in the unobservable entities as we do in the observable effects. That struck me as realism because it's a view on which the unobservable apparatus of scientific theoryies are taken to be real (in whatever sense observable things are so taken).
1[anonymous]11y
Voted lean toward Scientific Realism, but possibly confused. There's no difference between "observed" and "inferred". Also, some parts of our models "exist" others are just parts of models.

God: theism or atheism?

[pollid:83]

Note: I don't think most theistic philosophers would consider the simulation hypothesis to be a variant of theism.

That's because they say "theism" but they mean "traditional religion". They probably wouldn't accept a reification of Azathoth either, or the Flying Spaghetti Monster.

4drnickbone11y
This is because the folks who take the simulation hypothesis as a serious possibility (like Bostrom) also believe we are in a non-interventionist simulation (one without a creator/controller who regularly intervenes to answer prayers, reward worship and so on). They don't seem to care too much about whether there are other simulations somewhere whose creators do intervene. Generally they'd concede the point if pressed (OK, somewhere in the universe of simulations and simulators there are gods, but not around here. Happy?) and then move on. The main point about "theism" (as opposed to "deism" say) is that God or the gods really affect us. They don't just exist as an abstract debating point: it actually matters to us that they exist.
3asparisi11y
If the being(s) running the simulation are all-knowing and all-powerful with respect to the simulation, I think most philosophers would go there, even theistic ones. Hell, if Spinoza's God counts as a theistic position, then the simulation hypothesis is practically golden.

"Gods are ontologically distinct from creatures, or they're not worth the paper they're written on." -- Damien Broderick.

Can anyone exhibit an actual theist who says that a Matrix Lord composed of non-mental, non-mysterious parts counts as a God? So far as I know this position is held solely by people who want to mock the Simulation Hypothesis.

Pretty sure the Mormons qualify. I'm not one myself, but I used to live next door to a few, and this looks like a fair representation of their beliefs. The money quote:

Mormons believe that human beings are children of God, and as such, have within them the potential to become like God. Got it? Let me say it more clearly. We believe that we can become Gods....Here’s shocker #2–we believe God used to be a man, just like us.

4adamisom11y
... which explains why there is actually a Mormon Transhumanist group, and why there was even a conference on the subject in April where I live (Salt Lake City; unfortunately couldn't attend)

Hi, I'm a simulation-theist. Nice to meet you.

8arundelo11y
If someone asks you whether you believe in a god, how likely are you to say "yes" versus "it's complicated"? (Or versus "no"?)
5Jayson_Virissimo11y
I'm quite likely to answer in the affirmative in most contexts. On the other hand, "are you religious?" would tend to get some version of the "it's complicated" answer.
3komponisto11y
I hold it in order to mock theism. Physicalist reductionism is such a fundamental part of my belief system that I think charity requires that theism be interpreted as the SH. I appeal to the popular perception of the absurdity of the SH in order to undermine professions of belief in theism.
0adamisom11y
How (do you use it to mock/undermine theism)?
-2komponisto11y
2adamisom11y
Cool. I should have specified 'I'm intrigued; can you move down a level of specificity (as to how)?'
2komponisto11y
Most people seem to believe that it's absurd to suppose we're living in the Matrix; I point out that theism is not significantly different from this.
1Jayson_Virissimo11y
According to my informal experiments with presenting the SA to friends, the absurdity reaction is highly sensitive to framing-effects and especially the particular sequence in which I present the premises.
2A1987dM11y
If the non-mental parts they are composed of are unlike the non-mental parts we are composed of, I would. I am made of quarks and leptons and things in Life are made of dead cells and live cells, and IMO that's more than enough for us to be ontologically distinct from us. The same would apply to the next level up (if there's one).
2asparisi11y
Well, hence the confusion about Spinoza's God. It's not really ontologically distinct for him, since his God 'is' the universe. (His God is also just mental-stuff, but Spinoza was an idealist, so to him all stuff was mental-stuff.) Now, Spinoza's might not count as a 'Matrix Lord' since he denies that God 'chooses' anything in our sense (he holds to a very strong version of the Principle of Sufficient Reason) so it's not like God is 'running' the simulation, as it were: God just is the simulation according to Spinoza. Likewise, most theistic philosophers are comitted to God being omniscient, omnipotent, and (usually) omnibenevolent. But some put that in 'absolute' terms while some put it in terms of this world: omnipotent regarding this world, etc. For the latter group, there is no clear philosophical dilemma in saying that a Matrix Lord counts as God, especially if that Matrix Lord is also the "First Cause." In fact, assuming that such a Matrix Lord exists, most theistic philosophers would be required to say it counts as God.

I just hit "Accept: Theism" by accident. Yes, accident, not divine providence, thank you very much. Is there a way to revote?

6jimrandomh11y
Currently no, there's no option to revote. I have a todo list which this is on, but it may be awhile as that todo list has way-cooler things on it like split-test polls, and I might not even get to any of it at all.
1adamisom11y
I nearly did too, which makes me wonder if a few people did; the only difference is an 'a' and I guess I assumed atheism would be at the top
1[anonymous]11y
I think it is not obviously wrong that we live in some slices of "all possible worlds", and within all possible worlds there are beings with fancy super powers or perfect moral authority. At the same time I recognize that the religious awe and devotion I have felt at times are quirks of my brain and not a result of causal interaction with those possible-world-thingies. But I'm not sure whether or not to label those thingies as gods / deities, in addition to calling them weird aliens to whom religion has no access. Finding out whether some of those thingies are simulating me wouldn't have any influence on my choice of label.

Laws of nature: Humeanism or non-Humeanism?

[pollid:80]

Humeanism: The laws of nature are compressed descriptions of salient patterns in the distribution of physical events.

Non-Humeanism: The laws of nature are not mere descriptions. They determine the distribution of physical events.

5DanArmak11y
Leaving aside any putative True Theory of Everything which we don't know yet, the laws we actually know and use today are definitely Humean. We should know, we made them that way.
0RobertLumley11y
I assumed the question was referring to the fundamental laws of the universe, which would be a theory of everything.
4DanArmak11y
A true theory of everything is by definition never wrong. In which case there's no observable difference between Humeanism and non-Humeanism, and it makes no sense to talk about the theory "determining" events or merely "describing" them. Define: theory of everything: maximally compressed, true and complete description of the physical evolution of the universe over time.
1RobertLumley11y
This was a point of some confusion to me. But "The laws of nature" to me means the fundamental laws of the universe, not the models we come up with. I dismissed my confusion as "oh, this must be another obnoxious thing that mainstream philosophy thinks."
0A1987dM11y
And “determine” too.
1Rob Bensinger11y
This wording suggests that a major distinction between Humeans and non-Humeans is that the former consider laws of nature 'descriptions' (i.e., linguistic/conceptual entities) while the latter consider laws of nature mind-independent. But we could frame the distinction either linguistically or metaphysically. Metaphysically, Humeans think that Nature's patterns are ultimately just coincidental recurrences, while non-Humeans think that observed patterns have some cause or explanation in a deeper unifying structure. I suspect that a lot of the support for Humeanism in the above poll can be explained by people pattern-matching 'laws of nature are... descriptions' to 'positivism / metaphysical humility,' whereas in reality Humeanism is just as substantive and speculative a metaphysical thesis as is non-Humeanism. As an aside, it should be noted that Hume himself was not a 'Humean' in the above sense. So the modern terminology is historically misleading here.
2pragmatist11y
I think contemporary Humeans would disagree with your characterization. They distinguish between accidentally true generalizations (like the fact that no human is over 10 feet tall) and law-like generalizations. Not all patterns count as laws, hence the use of "salient" in my definition. So a law is not merely a coincidental recurrence. Many Humeans believe that any particular instance of a salient pattern of recurrences is explained by the existence and salience of the pattern, so laws are explanatory. Also, a number of Humeans do cite metaphysical humility as an advantage of their account of laws. Unlike non-Humeans, who need an additional fundamental metaphysical kind besides spatio-temporally distributed properties (i.e. laws of nature), Humeans purport to make do with just the properties themselves, reducing the laws to the properties. If this works, then it does seem like a more metaphysically parsimonious account, at least according to some accounts of parsimony. Of course, the Humean account is still a metaphysical account, so it wouldn't be congenial to the strict positivist, I suppose.
2Rob Bensinger11y
Out of curiosity, do you have on hand any good articles on how Humeans (metaphysically, epistemically, etc.) distinguish 'mere patterns' from 'capital-p Patterns'? (Aside from anthropocentric concepts like explanatory reducibility.) 'Salience' is usually an anthropocentric / psychological concept. What is meant by it here? I ask because we have to be careful not to allow non-Humean 'salience-making' properties. If salience is something metaphysical that makes certain patterns objectively 'special' and 'lawful' and 'regular', then I'd normally think of salience as non-Humean. On the other hand, if 'salience' is a non-unifying brute fact ('brute correlation,' 'unexplainable recurrence', 'irreducible coincidence', etc.), or is somehow relative to our human projects and interests and standards, then I'd normally think of it as Humean. Does this jibe with your experience? Yes. And 'metaphysical humility' here means 'metaphysical commitment to a smaller domain of entities or kinds'. My point was just to hammer home that it doesn't mean 'avoiding doing difficult speculative metaphysics' or 'skeptically withholding judgment' or 'treating lawfulness as a human construct' or 'positivistically denying that non-Humean laws are intelligible'; the latter four options preclude Humeanism as much as they preclude anti-Humeanism, and my worry was that some positivism-friendly LessWrongers would misconstrue Humeanism as somehow unmetaphysical, rather than just as metaphysically sparse. (Though, again, this is complicated; non-Humeans claim that Humeanism is metaphysically profligate, say, because brute correlations that continue to recur, and recur, and recur are more startling than unitarily 'law-governed' recurrences. Cf. Eliezer's concerns with treating our universe as metaphysically first-order.) A side-note, since it hasn't come up: A lot of people in the Humeanism debate unpack the dispute modally. On this conception, Humeans consider the laws and regularities of nature cont
0pragmatist11y
The classic neo-Humean account is David Lewis's. It's described in the SEP here. I also wrote a post about it here. The account does appeal to the simplicity of descriptions, which is arguably a somewhat anthropocentric concept, but I see that as a feature, not a bug. The laws are ways of efficiently organizing our understanding of the universe, not fundamental mind-independent entities (see the last section of my post). I don't think Lewis would agree with my characterization in this regard, I should say.
8maia11y
Other: What's the difference?
7AlexMennen11y
Other: there are high-level laws of nature that are compressed descriptions, and low-level laws of nature that determine events.
[-][anonymous]11y110

That's non-Humeanism.

2AlexMennen11y
That makes sense. Non-Humeanism it is, then.
4Jayson_Virissimo11y
"Lean toward: non-Humeanism", because given that we are in a simulation, it should be possible to recover "laws" that are isomorphic to constants in the source code via reverse engineering (natural science). Whether Humeanism holds outside of our own "world" seems like a separate question.
1[anonymous]11y
Other: Both. Our laws are compressions of the distribution of physical events. There is some underlying rule set that determines the distribution of physical events.
1magfrump11y
If I believe that the reason we get nicely compressed descriptions of salient patterns, because the mathematical abstractions have concrete realizations in the symmetries at the basis of physics, is that Non-Humeanism? And if I further hold the propositional belief that using the words "The laws of nature are not mere descriptions." is likely to make me too attached to current beliefs about physics (or more likely evolutionary psych) and the belief that one can never have perfect knowledge, so any conception of natural laws I (or anyone) will ever have will be Humean, should I have hit "other"?

Time: B-theory or A-theory?

[pollid:79]

B-theory: Specifying the temporal ordering of all events in space-time exhausts all the objective temporal facts about those events.

A-theory: Specifying the temporal ordering of all events in space-time does not exhaust all the objective temporal facts about them. There is a further temporal fact about a given event: whether it is in the past, in the present or in the future. These are objective facts that are not fixed by merely specifying which events happen earlier or later.

6DanArmak11y
In the past, present or future of what? Of me, who is saying it? But that is merely a temporal fact of the first kind: the given event is in the past/future of the event of my saying whether it is in my past/future.
9pragmatist11y
The point of A-theory is that past, present and future are non-relational. There is an objective fact of the matter about which slice of the space-time manifold is the present, although this fact keeps changing, of course. So yeah, one way to think of the difference between A-theory and B-theory is that the B-theorist thinks of past, present and future as relational terms. Just saying "past" isn't enough, you need to specify what it is in the past of. But the A-theorist thinks it makes sense to talk of past, present and future simpliciter.
4DanArmak11y
An event (a point in spacetime) is really objective wrt time: that is, it is outside of time. The ordering of two events is also objective. The matter of where we are in time - and therefore, whether an event is in our past or in our future - is constantly changing as time flows. It is not a time-objective fact about an event that it is in our future, for tomorrow it will be a fact about the same event that it is in our past. So these facts about an event are subjective: they depend on when in time you are when you are making the judgement. Disagreeing with this seems like saying the word "objective" should mean two different things in the two above paragraphs. Which is apparently what almost all big philosophical arguments reduce to. Sigh.
2pragmatist11y
You're describing a B-theoretical perspective. An A-theorist will disagree with this characterization. Many A-theorists believe that the only objects that exist are present objects. So time isn't like space, where "here" is "here" rather than "there" simply because you happen to be "here" and not "there". You could have been "there", in which case "there" would have been "here" for you. Time, A-theorists claim, is different, because you couldn't be located anywhere (anywhen) else except in the present. It's not that the present is the present for you because you happen to be there; it's that the only time at which objects exist (and, by extension, you could exist) is the present. So it's not like there are other people who exist at other times, for whom the present is different. Don't ask me to defend this view, though, because I think it's nuts. A-theory vs. B-theory is one of those philosophical debates which should be declared closed. B-theory has clearly won.
0drnickbone11y
I don't think anyone's made A-theory consistent with special relativity. But you can bodge general relativity/cosmological models a bit to get something that looks a bit like A-theory by insisting on a preferred foliation (e.g. look at spacelike surfaces with constant proper time since the Big Bang, and count everything on one such surface as the "real now")
1crazy8811y
See Craig Borne "A Future for Presentism" for someone who claims to have made an A-theory consistent with special relativity.
1pragmatist11y
Bourne, from what I recall, makes A-theory consistent with special relativity by positing that there is an (undetectable) privileged reference frame. Is this correct? If it is, I wouldn't really call that "consistent with special relativity", more like "flying in the face of the central lesson of special relativity".
1crazy8811y
Yeah, that's basically right but I'm not sure that you're being entirely fair to Bourne. First, I think under standard definitions of consistent, he does show that an A-theory is consistent with SR (ie. they can both be true at once). Second, I'm not sure it's fair to say that he is "flying in the face of the central lesson of SR" - he may be flying in the face of the standard way of interpreting this lesson but his claim is precisely that this way of intrepreting the lesson isn't the only way. The lesson can also be interpretted as "no privileged reference frame can be detected." I agree with the basic anti-Bourne sentiment but wonder whether the wording of your response undermines his claims more than they deserve - I don't think they are inconsistent nor that they fly in the face of the central lesson. I guess I more just think that postulating undetectable physical features of the world in order to make a metaphysical thesis - that is based on intuition (the "present" intuition) that can be explained away - come out to be true is undesirable. Though it should be noted that Bourne's conclusion is mostly that presentism is the only sensible A-theory but that it's not possible to decide between presentism and a B-theory (that is, he doesn't argue that presentism must be true).
0drnickbone11y
So this is just like the "bodge" I described for general relativity/cosmology, but with even less justification? The preferred foliation has no physical motivation at all, and is plucked entirely out of the aether (so to speak). I'm not sure I'd count that as consistent with special relativity. Though in principle it would give the same predictions as special relativity.
3drnickbone11y
Here's maybe another way of thinking about it ... A theory: The English sentence "Today is a Thursday" has the same meaning on all days when it is spoken, but its truth-value varies depending on when it is spoken. (The sentence always corresponds to the same proposition, but that proposition has variable truth value). B theory: The English sentence "Today is a Thursday" means something different when it is spoken on different occasions. Each specific utterance has a truth value which never changes. (Again, each utterance corresponds to a different proposition, but any one such proposition always has the same truth value.) If you can live with propositions whose truth value changes with time, then you're probably an A-theorist. If something smells fishy about that, and you think that in any given possible world, a proposition is either true or false and never changes its truth value, you're probably a B-theorist.
2A1987dM11y
If put this way, definitely B. "Now" is a deictic, meaning 'at the time when I'm speaking', much like 'here' is a deictic meaning 'in the place where I'm speaking'.
2faul_sname11y
What happens when you throw relativity into the mix? How do you specify the order of events in space-time when it varies based on the location of the observer?
8pragmatist11y
In relativity you still have temporal ordering of events, but it is not a total order. In a Newtonian world, every event is either earlier than, later than or simultaneous with every other event. In relativity, some events can still be unambiguously described as earlier than or later than others. Event A is earlier than event B if it is in B's past light cone. There is no longer a relationship of simultaneity, however, and some pairs of events are not related by the earlier than/later than relation. The temporal order is a partial order.
0faul_sname11y
That makes sense. Although it looks like you either have all information regarding where/when all events occurred relative to each other (which would necessarily give information about where/when all events happened in relation to an observer) or you have a partial ordering, which means you don't have all the information. The more I think about this, the more confused I get. How exactly does A differ from the information of what is in the observer's past light cone and what isn't? Or is this based on the idea of there being a single, consistent "present time" throughout the universe?
4pragmatist11y
The partial ordering actually encodes a surprising amount of information about space-time. Specifying the temporal ordering relationship between space-time points also fixes the topological structure of space-time, its differential structure, and the metric up to a conformal factor. It doesn't let you reconstruct the full metric, and maybe you think that certain metrical facts count as "temporal facts" (which seems right), in which case the definition of B-theory I gave above should be modified to include the claim that those metrical facts are provided as well. I don't want to make the definitions too technical though, and I think the current version gets the essential idea across, so I'm going to leave it as is.
6AlexMennen11y
Partial ordering.
0[anonymous]11y
What? since when can you specify a global order of events?
0pragmatist11y
See here. Of course, we have to assume that our space-time meets certain causal conditions (it doesn't have closed timelike curves, for instance).

Lean toward B-theory if pushed to answer, but I wonder what cognitive algorithm even generated this as a possibly interesting question.

Also, who the hell has invented the names for that?

7A1987dM11y
I hate names likes that (incl. System 1 vs. System 2 thinking, Type I vs Type II errors, etc.). I can never remember which name stands for which thing.
5[anonymous]11y
J. M. E. McTaggart.
6Alicorn11y
And he didn't mean to name competing theories about time; he was trying to dismiss the both of them.
1[anonymous]11y
I think he was just a B-theorist (though he thought the self-contradictory A-theory was an ineliminable part of our thoughts about time).
5Alicorn11y
Wikipedia appears to confirm my memory of it ("McTaggart argued that the A series was a necessary component of any full theory of time, but that it was also self-contradictory and that our perception of time was, therefore, ultimately an incoherent illusion.")
0[anonymous]11y
Does it say he rejected the B-Theory?
4Alicorn11y
It seems like the sentence I quoted indicates that he didn't consider it a "full theory of time". I don't feel like rereading the actual paper, though.
7[anonymous]11y
Looking at the paper, I find that you're right.
1crazy8811y
From memory, McTaggart argues that change is a necessary part of our concept of time. That B-theories can't account for change and that A-theories are incoherent. Consequently, time does not exist (but presumably something with some similarities to time does)
2Rob Bensinger11y
Philosophy trivia: "J.M.E. McTaggart" stands for "John McTaggart Ellis McTaggart".
8Oscar_Cunningham11y
"Other": The nature of physics has a strong possibility of being such that the question makes no sense.
2[anonymous]11y
Same other, but with the nuance that there is probably a refinement of the question that does make sense. A rough, likely wrong attempt at a refinement: 1. B'-time: as before, but with a partial order of events; 2. A'-time: as before, but with the extra data coming from any observer who can specify those events that conclusively are in their past or future.
1magfrump11y
Sort of lean toward B-theory, but I suspect other temporal facts may exist. However, they are not of the form "X is in the present/future/past." Also, the nature of time may make these the wrong questions to ask as noted below.
0diegocaleiro11y
Other: Events might not be ordered, yet time exists. This would be the case with the Timeless Barbourian Physics Yudkowsky discusses in the sequences. Also could be the case if some variety of many worlds of quantum style, or all possible worlds of Lewis style is the case. The Anthopic folk, like me, Katja, Bostrom and several others, might also amuse themselves by inquiring: Could in one of the many infinite universe (or Big universe) theories be the case that all time-like facts are some form of indexical fact?
0asparisi11y
Other. I think the most fundamental aspect of time is local quantum instability, rather than relationships such as "earlier than/later than" or "past-present-future" between events.
0AspiringRationalist11y
Other: I don't know. Poll: Should these polls have an "I don't know" / neutral option? [pollid:95] Note: I haven't included "I don't know" on this poll because its purpose is to get the balance of opinion for/against. The other polls in the comments on this article are intended to get an overall picture of the local population, so it would be interesting if a question had a high percentage of "I don't know" responses.
-8shminux11y

Perceptual experience: disjunctivism, qualia theory, representationalism, or sense-datum theory?

[pollid:105]

9pragmatist11y
Disjunctivism: In normal cases, when a person is perceiving something, the object of their perception is a mind-independent object. The character of the person's phenomenal experience is explained by the properties of the object (for instance, a person perceiving an apple has an experience of redness because the object of their perception -- the apple -- is red). However, when a person is hallucinating or experiencing an illusion, the object of their perception is some sort of mind-dependent entity (perhaps sense-data; see below). So hallucination and veridical perception are substantially different kinds of mental processes. Representationalism: Perceptual experience is representational. It represents our immediate environment as being a certain way. Since representations can be both accurate and inaccurate, we can understand both veridical perception and hallucination as the same kind of process. The difference is merely in the success of the representation. To be perceiving is to be representing one's immediate environment in certain ways (visually, aurally, etc.), and perception is accurate to the extent that this representation corresponds to reality. Sense-datum theory: The objects of our perception are not mind-independent entities, they are mind-dependent objects called sense-data. These are objects like "a red spot at such-and-such position in my visual field". We infer the existence of mind-independent objects from patterns in the sense-data we perceive. Qualia theory: [I don't think I fully understand the claims of qualia theory. I have tried to describe the kinds of things qualia theorists say, but if it appears confused that's because I am confused.] The phenomenal character of our perceptual experience (the particular way our experience feels) is non-representational. We don't infer information about the external world from the particular feel of our conscious experience; rather, our conscious experience is simply what it feels like (from the inside
4Swimmy11y
I wonder how many disjunctivists have actually taken hallucinatory drugs?
3TheOtherDave11y
Just to be clear: sense-datum theory is not asserting that distal stimuli don't exist or that we're unjustified in inferring their existence or otherwise making a claim about existence. It is merely asserting that we infer their existence from sense data, as opposed to, um.... doing something other than that. Yes?
2pragmatist11y
Yes, the claim is that mind-independent objects are not the direct objects of perception, sense-data are. In so far as perception gives us information about distal objects, that information is inferred from patterns in sense data.
2arborealhominid11y
I'm not totally clear on the distinction between representationalism and sense-datum theory. Do you think you could explain it in a bit more detail?
0Peterdjones11y
I think qualia theory is quite othogonal to (in)direct realism. Different qualia tbeorists hold qualia to be direct perceptions of external properties, or to be qualitites of sense data, or to be adverbial. Etc.
0pragmatist11y
Then in what sense is it a distinct view about perception? Are the poll options just poorly formulated?
1Peterdjones11y
Yes. Sense dataum theory barely differs from representationalism, direct relaism isn't mentioned, and qualia theory is orthogonal.
1pragmatist11y
I'm not sure I agree with your assessment. Sense datum theory is significantly different from representationalism, since sense data aren't supposed to intentional objects. Disjunctivism is the only form of direct realism anyone takes seriously any more, as far as I'm aware. You may be right about qualia theory, but I have heard people use the term as a stand-in for adverbialism. I suspect that's what it's supposed to represent in the poll.
0Peterdjones11y
It looks like the list of options was cribbed from the The SEP article , but without much context.
0torekp11y
For someone who isn't confident of understanding qualia theory, you stated it quite well. Near the end it could use a tweak, I think: "my conscious experience in its entirety isn't essential to my knowing that the apple is red. The phenomenal quality of the experience is just an effect ..." This leaves the verbal issue open, of whether perceived facts as well as phenomenal qualities count as part of "conscious experience".
-2diegocaleiro11y
I don't mean to be rude to the fellow, but my current understanding of why Chalmers (main Qualia theorist) says qualia are what he says they are is isomorphic to Lee Smolin's critique of string theorists. They become string theorists because of sociological reasons. It is the part of physics in which high intelligence is recompensated faster. Chalmers took a polarized view so that the rest of what he defends became visible. It worked fantastically well. http://lesswrong.com/lw/58d/how_not_to_be_a_na%C3%AFve_computationalist/ EDIT: in the link above I suggest a reading of the article "The Content and Epistemology of Phenomenal Belief" by Chalmers. It is the one in which this possible case is most easily visible. It is the case in which inconsistencies in the Chalmerian definition are most visible.
3ModusPonies11y
Other: all of these seem like plausible models with a nonzero amount of predictive power. I don't see how the truth of one of these implies that the others are false. Representationalism and sense-datum theory seem the most useful, I guess, but my real answer is "I don't know, ask a neurologist."
1BrassLion11y
I'm not sure how you would even create a test to distingush these models. I'm not sure if my understanding is incomplete or if that's a warning sign that they aren't different or aren't coherent.
2Eugine_Nier11y
Other: These are all true for different meanings of "the object of one's perception".
0Peterdjones11y
Ill-formed question. See my other comment.

Moral judgment: externalism or internalism?

[pollid:85]

Externalism: It is possible for a person to sincerely hold a moral belief (or make a moral judgment) without feeling any motivation to adhere to that belief/judgment. The claim is not just that the motivation might be trumped by other motivations, it is that it is possible for there to be no motivation at all.

Internalism: It is impossible to sincerely make a moral judgment without being motivated to act in accordance with it, although it may be the case that the motivation is trumped by other countervailing motivations.

Other: This is a not-very-interesting definitional question as to exactly which kind of mental states should be counted as "sincerely making a moral judgement".

General defense of the above type of reply: Voting "Other" on questions that seem to you confused or seem to turn on irrelevant matters of small definitions, rather than making up a definition and running with it, etcetera, is probably a good barometer of LW-vs.-philosophy opinion.

The subject matter of humanity::morality is a mathematical object which Clippy could calculate, if it ever had any reason to do so, which it wouldn't, but it could, without being at all motivated to do anything about that. However, if "morality" is being given an agent relative definition then no, whatever you're not motivated to do anything about, even in the slightest, doesn't seem like it should be called Alejandro::morality.

7thomblake11y
I doubt it. In my experience, if you allow a "Please specify" answer, philosophers will pick that for practically any distinction.
7[anonymous]11y
It is, at any rate, if you have some evidence that philosophers (professional? historical? what?) make up definitions and run with them when they don't understand a question. Otherwise, it's probably a very, very bad barometer.
0Nisan11y
I agree with this, even though I voted "externalism".
3drnickbone11y
Don't psychopaths count as evidence for "externalism"? (They know what they're doing is wrong but don't care.)
5TheOtherDave11y
Not in and of itself. Suppose a psychopath says "I know killing people is wrong but I don't care" and kills someone. Wearing an externalist hat, I say "See? They made a moral judgment, but it doesn't constrain their behavior." Wearing an internalist hat, I say either "Maybe it did constrain their behavior, just not enough to prevent them from killing someone, because other factors motivated their behavior that aren't present in non-psychopaths" or "Maybe it wasn't a sincere moral judgment, they were just echoing what they've been told, like a blind person saying stop signs are red." That said, I can imagine well-designed experiments that would be evidence for one or the other. E.g., if we identify what parts of the brain are generally engaged during moral judgments, and what parts of the brain are engaged during recitation of memorized facts, and determine that when psychopaths say "killing people is wrong" the latter brain areas are engaged but the former are not, I would consider that evidence that psychopaths don't in fact make sincere moral judgments when they say that.
6Richard_Kennaway11y
We know from the effects of morphine that it is possible to experience "pain" without it "hurting". (I wonder if any philosopher foresaw that this is even possible?) Given that, it is quite conceivable to me that a psychopath might experience the feeling of "moral wrongness" without the motivation "I shouldn't do this". Maybe that isn't what's going on, but no process of reasoning about morality can rule it out. Philosophers, even materialist ones, are apt to think of the mind as being some sort of logical entity, about which they can prove that certain mental behaviours are impossible. But when the brain goes funny, all of those arguments fail. Which implies that when the brain hasn't gone funny, the arguments still don't prove anything, because how do we know that even a normal brain doesn't do other "absurd" things? Only by observing how it actually works, not by any process of pure reason. Compare "akrasia". How can you want to do a thing, have the ability, and yet not do it, all the while bemoaning the fact that you aren't doing it? Obviously absurd, impossible, a contradiction. But the fact that people do go funny in this way is so familiar to everyone that philosophers can't get far by arguing that it can't happen.
2TheOtherDave11y
I certainly agree that we can discover things about the world that make us realize that some phenomenon P that we naively thought was indivisible in fact turns out to have internal structure, such that an event can demonstrate some-but-not-all-of P. I'm mostly of the opinion that when that sort of thing happens, the best move is to get very clear about what level of abstraction we're talking about, what concepts apply at that level, and how we refer to those concepts. Sometimes it's useful to talk about "cells", but if I'm interested in how mitochondria interact with the endoplasmic reticulum the concept to which "cell" refers isn't useful to me; I need new concepts (and it's probably best if I don't assign them homophonic labels). Are people on morphine actually experiencing pain? Yes, they are. No, they aren't. It's mostly not a helpful question. They are experiencing some properties of pain and not others, and what we've discovered is that our normal experience of pain has separable internal structure. So far, so good. And, sure, when an actor playing Oedipus screams and yells about how painful having his eyes plucked out is, he's experiencing some of the properties of pain, and it can be interesting to consider which properties those are. But I can also jump up a level and ask whether the actor is really experiencing pain, and on balance my answer is pretty unambiguously "no." (Though I'm prepared to make exceptions for certain actors and performances, given enough counterevidence.) Similarly, if I experience akrasia, that tells me that motivation has internal structure, and I can experience some of its components (e.g., the belief that I want to do something) without others (e.g., the actually-doing-it). At a lower level, I can usefully consider the relationship between those components. At a higher level, I can ask whether I'm really motivated to do it. (My usual answer is "no." I understand that judgments vary on this one.) So, OK. I agree that moral jud
5drnickbone11y
But a blind person can still sincerely say that "stop signs are red". Their justification for saying so may be different from a sighted person's, but the statement is still sincere (the blind person really believes it). Is it part of the internalist claim that it is just impossible to acquire moral knowledge by such third-party means? For instance, simply observing what other people say about "right" and "wrong" and building an inductive concept about what actions the words describe, but without any emotional preference for "right" over "wrong"?
2TheOtherDave11y
I can't speak for internalism, but it certainly seems to me that what I'm doing when I say that it's wrong for me to eat pork by the standards of Judaism is different from what i would be doing, were I an observant Jew, if I said it was wrong for me to eat pork.
3drnickbone11y
But still, how does this break the analogy with "red" for a blind person? For instance, a blind person could sincerely believe all of the following: "Stop signs are red" "My evidence for believing that stop signs are red is that sighted people have told me so" "Red things aren't red just because people say they are. Rather they have some inherent property to do with the way light reflects off them which causes people to describe them as red" "Some religious sects call certain signs red which everyone else calls orange, but these sects are wrong; the signs are in fact orange. Members of the sects have been brought up that way, and are sincere in their beliefs, but they are still wrong" "If I were a member of such a sect, I'd probably also believe that the orange signs are red" In short it seems possible to have a complete and sincere set of beliefs in "red" as an objective world property, without ever seeing red yourself or ever having any emotional response to it. I don't immediately see why the same can't be true for moral beliefs.
2TheOtherDave11y
Fair enough. It's certainly true that there are two different things being done by psychopaths and nonpsychopaths in the original example, but it might be that both of those things count as genuine moral beliefs, just as the two different things involving a stop sign can count as genuine beliefs about color. OTOH, it might be that only one of them counts as a genuine moral belief, just as only one of the things involving a stop sign counts as a genuine perception of color.
3magfrump11y
I accept externalism on the part of some possible sentient creatures, but I think this question with respect to humans is a question of definitions, and even if you decide on a definition it becomes an empirical question I don't know the answer to.
2[anonymous]11y
Other: I was unable to resolve my thoughts on this at all, and am not sure to what degree hearing more arguments either way would help that.

Is there somewhere a glossary for all the questions? That would be very helpful (beyond this survey).

Also - there was already a similar thread:

http://lesswrong.com/lw/56q/how_would_you_respond_to_the_philpapers_what_are/

The comments have some answers (though not in a convenient machine readable form).

3Jayson_Virissimo11y
pragmatist has added comments with descriptions of the key terms to most of the poll question comments. Interesting; perhaps someone would be willing to check them against this newer data for changes in Less Wrongers' beliefs over time.

A few problems with this LW survey:

  1. Most of the interesting options in the original PhilPapers Survey are collapsed into 'Other'. This makes it needlessly tempting to side with one of the named positions in order to make one's answer usefully contentful. It also makes our comparisons to the original poll much cruder. The original survey provided (regularly used) options for: 'accept all', 'reject all', 'accept an intermediate view', 'accept an alternative', 'the question is too unclear to answer', 'there is no fact of the matter', 'insufficiently familiar

... (read more)

Zombies: inconceivable, conceivable but not metaphysically possible, or metaphysically possible?

[pollid:109]

A zombie is physically identical to a human being but does not possess phenomenal experience. There is nothing it is like to be a zombie.

Inconceivable: We cannot fully conceive of a zombie. If you think you have a coherent conception of a zombie, it is because you haven't thought about your conception carefully enough. Sufficient thought will reveal that your conception is incoherent.

Conceivable but not metaphysically possible: One can arrive at a coherent conception of zombies, but objects that match this conception cannot possibly exist, not even in worlds with different laws of nature than ours.

Metaphysically possible: The existence of zombies is possible.

3TheOtherDave11y
Given these options, I think I have to reluctantly choose "metaphysically possible." A decent respect to the opinions of Less Wrong requires that I should declare the causes which impel me to this. I have never been able to conceive of a zombie and a physically identical non-zombie without subsequently concluding that my conception leads to positing causeless effects (e.g., a zombie spontaneously talking about its nonexistent phenomenal experience while using all and only the cognitive structures that the non-zombie uses to talk about actual phenomenal experience). But, well... causeless effects aren't incoherent, or metaphysically impossible, they're just vanishingly unlikely. . (Not even necessarily that. E.g., in a world where there are only a few possible utterances, one of which, X, is conventionally understood to be a reference to the speaker's phenomenal experience, it's not even all that unlikely for a zombie to just kind of coincidentally happen to utter X in exactly those circumstances that a non-zombie analog utters them due to phenomenal experience.) Further, nothing in these definitions asserts that humans do have phenomenal experience, which allows for Dennett's answer ("We're all zombies.") In which case cause and effect aren't even at issue... all of us zombies just happen to talk about phenomenal experience despite not having any, because our brains are wired to do so, presumably for signalling reasons but possibly because the Matrix Lords think it's funny, or for other reasons. This strikes me as implausible... I mean, it sure does seem to me that I have phenomenal experience, and I can't imagine being wrong about that. But that might just be a failure of my imagination. Which seems to add up to "zombies are metaphysically possible" on this account. I'm not quite sure.
4pragmatist11y
How did you answer the physicalism question? If you think any physical duplicate of our world must also be a mental duplicate then I think you're committed to the impossibility of zombies. Think about it this way: Do you think there can be a world where the distribution of micro-physical properties is identical to ours but which does not have tables? Presumably no, because you believe that once microscopic properties are distributed a certain way in some location, you can't avoid having a table in that location. But you don't think the same is true of phenomenal experience if you believe zombies are possible. You think that there needs to be something more than just the right distribution of physical properties for consciousness to appear. I suspect you're conflating epistemic and metaphysical possibility when you say causeless effects are vanishingly unlikely. Take the sentence "2 + 2 = 5". I think its metaphysically impossible for that to be true (assuming the meanings of the terms are kept constant) -- there is no possible world in which it is true -- but I don't assign it probability 0 because (as Eliezer has pointed out) I acknowledge that there is some possible sequence of experiences that might lead me to believe it (or increase my credence in it). Thinking something is impossible does not mean assigning it probability 0. Dennett doesn't think we're all zombies, BTW. He's skeptical of qualia as a coherent concept, but he doesn't deny that we have phenomenal experience.
2TheOtherDave11y
I don't believe mental events have nonphysical causes, which is precisely why I consider zombies to entail causeless effects. And yeah, perhaps I'm just reading too much into "metaphysically impossible" as distinct from "impossible"; the truth is I really don't know how to think cogently about what would be impossible if the laws of nature were different. The "we're all zombies" bit was intended with tongue in cheek; it's actually a direct quote of his from some book or another -- Consciousness Explained, probably -- that I read like 20 years ago and stuck with me. He isn't entirely serious when he says it, of course, and IIRC has a little footnote that says "To quote this phrase out of context would be the height of intellectual dishonesty." or words to that effect. EDIT: That said, another relevant Dennetism that stuck with me was his response to an undergrad at a seminar I was listening in on years back. The undergrad said, in effect, "But I don't feel like a merely computational process!" and he replied "How do you know? Maybe this is exactly what a merely computational process feels like!"
-1thomblake11y
On reflection I agree. "Metaphysically impossible" is a rather strong requirement if taken strictly, and I think strictly is the right way to take it when talking about metaphysical possibility. So if you think there's a possible world where human actions just happen by chance to line up with some mysterious experiences and zombie actions don't, then it's metaphysically possible. I'm not sure whether Dennett's answer should really be read as "zombies are metaphysically possible" but I'm now convinced this whole question is silly.
-1TheOtherDave11y
Well, yes.
1A1987dM11y
What is meant by “conception” exactly? Because that doesn't appear to make sense to me.
0Rob Bensinger11y
Went for "metaphysically possible". I think zombies exist, and we are they.
0[anonymous]11y
Other: I wouldn't say it's inconceivable exactly, but I think the thought experiment, in framing the possibility of a decoupling of phenomenal experience from cognitive architecture, primes for some intuitions that don't make a lot of sense to me.
0drnickbone11y
Went for "conceivable but not metaphysically possible". It seems pretty clear to me that we can and do "conceive" of zombies, or of other puzzles like inverted spectra (you see red while I see green, but we're both looking at the same tree, and both call the tree "green"). However, this is because of a mental trick in the way we conceive mental states. Basically, for normally-sighted people, it seems that when we imagine a physical object (like a tree), we imagine it from an external perspective. We induce in ourselves a mental state similar to the one we would get from looking at the object concerned (the tree). However, when we imagine a mental state itself (such as an experience of green), we do so sympathetically, or from an internal perspective, by inducing in ourselves the same or a similar mental state. So this allows us to "mix and match" brain states and mental states. For instance, I can imagine your brain while you are looking at a tree (I have a mental picture of grey matter with lots of neurons firing) while at the same time imagining the mental state of experiencing red. No problem at all... it seems perfectly conceivable that these could happen together; instant inverted spectrum. Or I can imagine the same brain state (same grey matter, same firings) while not imagining a mental state at all (it's all neurons firing in complicated chains, but no-one's at home)... again seems perfectly conceivable that these could happen together; instant p-zombie. But it's all just a dumb quirk of imagination. Because of the different imaginative techniques, we fool ourselves into conceiving apparent possibilities that aren't really possible after all. One way to debug the imagination is to ask ourselves this: "Suppose I were forced to imagine a mental state from the external perspective, the same way I imagine a tree. What would that look like?" And the short answer is "Well, without scientific evidence, I really don't have a clue. I have no intuition whatseover

Truth: correspondence, deflationary, or epistemic?

[pollid:108]

9pragmatist11y
Correspondence: A proposition is true if and only if it bears some sort of congruence relation to a state of affairs that obtains. When I say that "P is true", where P is some proposition, I am saying that P stands in this relation to some portion of reality. Deflationary: Ascribing truth to a proposition amounts to no more than asserting the proposition. It does not mean you are attributing some further property to the proposition (such as congruence with some state of affairs). Saying "P is true" is the same as just saying "P'. Epistemic: To say that a proposition is true is just to say that it meets a high standard of epistemic warrant, and that we are thereby justified in asserting it. The search for truth and the search for justification are not separate goals. There is no more to truth than sufficiently powerful justification.
2Richard_Kennaway11y
"Deflationary" is not the same sort of thing as the other two theories, either of which one might believe while being a deflationist. In fact, I don't see a reason to think deflationism is false. What is actually meant by "just saying P" is not answered by deflationism, but by an actual theory of truth. And I still think that after consulting the Stanford Encyclopedia on the subject.
4pragmatist11y
The whole point of deflationism is that there is nothing further to be said about truth other than what I said in my description. They think of truth as a shallow notion that plays no significant explanatory role in our accounts of language and meaning. If you think that the deflationist claim needs to be backed up by a more substantive theory of truth then you are not a deflationist. So there you have it, a reason to think deflationism is false. You can think of deflationism as an anti-theory of truth rather than a theory of truth. The central claim is that no theory of truth is needed. The linguistic function of truth claims can be understood in terms of the formal disquotational scheme I outlined, and there isn't really any deeper metaphysical question about what Truth is. If you ask a deflationist, "What do you mean by 'just saying P'?", he might respond with a theory of meaning -- a theory that gives a systematic account of how sentences are assigned semantic content -- but he will not respond with a theory of truth.
2Richard_Kennaway11y
It seems to me that if a deflationist answers the question "when can you correctly say "snow is white"" with the answer "when snow is, in fact, white", then the deflationist is a correspondencist. If he says "when you have good enough evidence that snow is white", he is an epistemicist. Deflationism on its own is surely just a shallow concern with the word "truth". Of course asserting "it is true that P" is the same as asserting P, but that is not an interesting fact. Is deflationism historically a response to some obsolete idea about "truth" as an immaterial substance that adheres to true propositions, by virtue of which they are true? Neither of the other two theories assert such an idea.
0thomblake11y
I totally agree.
0Cthulhoo11y
I'm honestly confused about this definition. Does it mean that once we define what truth is, we can then assert if a proposition is true or not ( truth and "justification" are in this sense intertwined)? In this case, isn't this just a broader case of Correspondence, or better, isn't Correspondence just a particular subcase of Epistemic?
6pragmatist11y
Perhaps an example of an epistemic theory of truth will help. Suppose you have a scientific community grappling with a certain set of problems. As their inquiry proceeds, they solve some problems and uncover new ones. In the process, they also greatly refine their methods. Imagine they eventually come to a stage where all their problems have been solved according to their own standards of warrant, and no open problems remain. One theory of truth (due to Charles Peirce) says that their beliefs at this stage of inquiry are true by definition. Truth is just what a community will arrive at at the ideal end of inquiry. How is this different from a correspondence theory? Well, the correspondence theorist would say that even though the scientists have fully resolved their inquiry according to their own standards, and it seems like their beliefs are highly justified, it still might be the case that they got it wrong. It still might be the case that their beliefs don't correspond to reality, in which case their beliefs are false. For Peirce, this claim makes no sense. According to him, what it means for a belief to conform to reality (or to be true) is for it to be a belief held by a community at the ideal end of inquiry. If we allow that our beliefs can be as justified as we could possibly make them and still fail to be true, then truth becomes a potentially unattainable goal, and for Peirce this would make truth a philosophically useless concept. The epistemic theory of truth is generally held by philosophers who are skeptical of metaphysics, and who think metaphysical concepts are only valuable to the extent that they make a difference to our lives. If there is a metaphysical distinction that does not correspond to a distinction in what we observe or how we should behave, it should be discarded. This is why they think the theory of truth should be closely tied to our epistemic practices, the stuff we do when we are trying to find the truth.
0TheOtherDave11y
OK, so... suppose this community then encounters some new experiences at time T that cause them to reject the solution (S1) to a problem previously considered solved, which they then re-close (with a different solution S2). A correspondence theorist wasn't sure before T whether S1 was true or false, they're still not sure after T whether S1 is true or false, and the same goes for S2 (and, for that matter, anything else someone might assert). Presumably they will also make statements like S1 was justified before T and unjustified after T, that S2 is justified after T, and might disagree about whether S2 was justified before T. An epistemic theorist asserts that S1 was true before T and is false after T, that S2 is false after T, and might disagree about whether S2 was true before T. A deflationary theorist... um... is not committed to any particular position about the truth, falsehood, or indeterminacy of S1 or S2 at any time. Yes?
0pragmatist11y
For Peirce, an end to all problems (and therefore an end to inquiry) includes the claim that there are no reecalcitrant experiences. It isn't just a temporary end to problems, it's a permanent end. This is why he refers to this as the ideal limit of inquiry. So the situation in your first paragraph wouldn't apply. Peirce specifies things this way precisely to avoid the consequence that truth-values can change with time. I should say, though, that this is only one specific epistemic theory, and an early (and therefore kind of unsophisticated) one. I chose it for ease of exposition.
0TheOtherDave11y
Well, OK, but then it seems Peirce's conception of truth is just as potentially unattainable (and thus philosophically useless, by his own account) as the correspondence conception. Unless I've reached an ideal limit of inquiry -- which of course I can't really know I've done, and am unlikely to have done -- then it seems I don't actually have truth, even on Pierce's account. That aside, though, point taken about epistemicism != Peirce; presumably if I actually care about the latter I should just read Peirce. I'm just being intrigued by it. I do think I roughly understand the concept now; thanks for the explanation.
0Cthulhoo11y
Many thanks to you and Vaniver, I have a clearer picture now (though this mostly looks like a debate over definitions, that can basically be solved by tabooing the word "truth").
3thomblake11y
When you've reached this point, you understand a philosophical debate, for most 20th-century philosophical debates.
0pragmatist11y
Not sure what you mean by this. The three theories of truth are different attempts to taboo the word "truth".
1Cthulhoo11y
Isn't this the point? Tabooing "truth", one can see that the theories really speak about (slightly) different concepts. Going back to your previous example, if one theory claims the scientists have reached the truth and the second doesn't, how does it change the reality? You can easily define some new words to correspond to the different concepts, and refer to the appropriate label under the different circumstances.
6metaphysicist11y
Then, you would merely choose which of the concepts is the one needed for a particular theoretical purpose. Right? Wrong! The arguments go to the concepts' coherence. This is why it's philosophy, not lexicography. For example, a correspondence theorist generally argues that the notion of an epistemological limit to which scientific findings converge need not exist and can never be established empirically. If correspondence theory is true, you aren't allowed to use the Piercian limit. It's a vacuous concept. Or, the correspondence theorist argues that the epistemological limit of scientific investigation can't even be defined without assuming a correspondence variety of truth (which the Piercian, in turn, argues can't exist). The correspondentist argues that if you define truth at a limit, then you have to define the truth that science is converging as itself the result of a scientific investigation at an endpoint, and similarly for the concepts you use to define scientific investigation, etc. Thus, a Piercian view, it's contended, produces an infinite regress. It's possible that both concepts are coherent, but that too would require a philosophical argument--and it's an unlikely result here, at least in my opinion: it's probably more likely that both concepts are incoherent than that both are coherent. These kinds of conclusions, philosophical and lacking in direct application, help inform the priors one assigns to just about every scientific controversy.
1Cthulhoo11y
Ok, this starts to sound more interesting, thank you for the reply. I tried to briefly google for "Piercian limit", though and it didn't turn out anything relevant. Any quick reference?
2metaphysicist11y
Theories using Piercian concepts are today usually termed antirealist or instrumentalist.
0Cthulhoo11y
Thank you, this is turning out a lot of material that I will definitely read.
0TheOtherDave11y
(blink) If I accept acorrespondence theory of truth, it seems that correspondence theory is not the sort of thing that is allowed to have a truth value. And if I reject a correspondence theory of truth, then I ought not believe that correspondence theory is true. So it seems that "correspondence theory is true" is necessarily false. No?
1metaphysicist11y
That's an excellent argument if it's the case that correspondence theory is not the sort of thing allowed to have truth values under correspondence theory. Why do you say it's not?
0TheOtherDave11y
Well, using pragmatist's cited definition of correspondence theory, a proposition is true if and only if it bears some sort of congruence relation to a state of affairs that obtains. What state of affairs is "correspondence theory is true" congruent with? I can't think of any. If you can, I'll happily be convinced my argument doesn't hold, but basically it seems to me that correspondence theory lays out a framework for thinking about truth, just as governmental constitutions lay out a framework for thinking about law. Correspondence theory itself is no more true (or false) than constitutions are legal (or illegal).
1metaphysicist11y
The concept of scientific truth--the concept used by scientists--is the state of affairs some correspondence theories purport to be congruent with.
0TheOtherDave11y
I might still want to know which concept I'm using when I say something is true. I might also want to know which concept you're using when you say something is true. Sure, if I don't know that (or am not confident I know that) then we can taboo "truth," but that gets unwieldy; if we can agree on a shared referent then communication is more efficient. Tabooing key words is sort of like running code under a debugger... a great way of identifying points of failure, but not the way I want to live my life. If I know whether someone's conception of truth is correspondist or epistemic, they can say "X is true" and I know what they mean about X without having to taboo "truth".
0Cthulhoo11y
This is something I understand and I can agree with. But it's a very practical problem, like making Europeans and Americans agree on the meaning of the word "football". It's very likely that I'm still missing something, though (see metaphysicist reply).
3Vaniver11y
Typical examples of the difference between correspondence and epistemic have to do with people being right on accident. For example: Is Bob's statement true? Is it a justified true belief? Correspondence would say that Bob did tell the truth, but epistemic would say that Bob's statement, though true, was unjustified.
6fubarobfusco11y
The philosophers call these Gettier problems. This particular story introduces what seems like it should be an extraneous detail: Bob, in saying he doesn't carry cash, is intending to deceive. But our everyday concept of truth is related somehow to honesty; the truth is what a well-informed and honest person would say. Bob here is being dishonest (he wants the mugger to believe something Bob thinks is false) and misinformed, so his dishonesty fails at his goal of protecting his cash. This goal is already lost. Knowing that, the question "but is his statement true?" seems to be unneeded essentialism.
2Vaniver11y
My impression is that this is what the whole debate is about. What matters to Corresponders is whether or not statements describe reality. Bob's statement correctly predicts whether or not he has cash on him- and "correct prediction" is their standard for truth. They wouldn't care that his dishonesty and misinformation cancel each other out, but would agree that in general dishonesty and misinformation lead to less correct predictions.
1drnickbone11y
I think there might be a difference here between truth of a proposition and truth of a sentence. Ascribing truth to a proposition looks the same as just asserting the proposition (deflationary account). Ascribing truth to a sentence looks like a combination of mapping it onto a proposition (interpreting the sentence) and then asserting that proposition. Since I would regard propositions and states of affairs as either the same, or isomorphic, this looks like the correspondence theory of truth for sentences.
1Benjy Forstadt5y
I strongly agree. This has been discussed in the literature, I believe. For instance, it comes up in discussions of the semantic paradoxes. There is no paradox involved in “disquotation” for propositions: the proposition that p just is the proposition that it’s true that p. When you’re talking about sentences though, you run into the liar paradox if you say that “p” is true if and only if p.

External world: idealism, skepticism, or non-skeptical realism?

[pollid:102]

Non-skeptical realism: A mind-independent reality exists, and we have epistemic access to its structure. We can acquire substantial knowledge about reality.

Skepticism: A mind-independent reality exists, but we lack epistemic access to it. We cannot know the nature of reality. We only have access to how things appear to us, and we should take seriously the possibility that this is very different from how things actually are.

Idealism: Reality is not mind-independent. It is either wholly or partly mentally constituted. We can know about reality because there is not much (or no) distance between how things appear to us and how things actually are.

4drnickbone11y
I'm not sure how well this fits with fallibilist accounts of knowledge (e.g. probabilism, Bayesianism). A Bayesian doesn't "rule out" possibilities when setting probabilities strictly between 0 or 1, so this technically looks like "skepticism". But if I claim that I'm 99.9999% certain that a mind-independent reality exists and I have substantial knowledge about it, that really doesn't sound very skeptical!
0pragmatist11y
You can read "rule out" as "no longer take seriously". The probability of a hypothesis doesn't have to go down all the way to 0 before I stop taking it seriously. I've edited the original description to reflect this.
1drnickbone11y
Thanks for this clarification. I was going for "lean towards non-skeptical realism" but would say "accept non-skeptical realism" under your new formulation. I don't rule out a simulation hypothesis, for instance, but can't say I give it serious probability weighting. (Bostrom considers it one of three disjuncts, and I can give reasons to assign the other disjuncts much higher probability.)
0A1987dM11y
Exactly what I was going to say.
2Rob Bensinger11y
Three problems: 1. This seems to entail the absurd proposition "p, but we have no way of knowing that p". I.e., it's not clear how to cash out 'epistemic access' in a way that allows us to know that there is a mind-independent world, without knowing anything further about that world. This uncharitably commits skepticism to an internal tension, if not an outright contradiction. 2. "We only have access to how things appear to us", inasmuch as it implies "We have access to how things appear to us", is itself a substantive doctrine about how reality breaks down, and one skepticism need not endorse. So this uncharitably assigns certain doctrinal commitments to skeptics as a group. 3. This reading assumes that skeptics are realists of some sort, or that they privilege realism as a hypothesis over idealism. The original question does not state this, so idealistic or neutral skeptics may be unfairly biased by this interpretation. 'Mentally constituted' is vague. If this just means that part of reality is mental (or irreducibly mental), then it seems to treat dualism as a form of idealism, which is very nonstandard.
0pragmatist11y
Fair enough. I should have said something like: "A mind-independent reality might exist, and if it does then we lack epistemic access to it."

Trolley problem (five straight ahead, one on side track, turn requires switching): straight or turn?

[pollid:93]

Trolley problem: There is a trolley traveling along a set of tracks. The driver has lost control of the trolley. On the track ahead of the trolley are five people who cannot get off the track in time and will all die if the trolley gets to them. You are standing next to a lever that can switch the track the trolley will take, preventing the deaths of the five people. On the other track is a single person who also cannot get away in time and so will die if you switch the track. Do you refrain from switching the track ("straight") or do you switch the track ("turn")?

3DanArmak11y
Addendum: you are not allowed to explicitly consider e.g. future reputational effects of your choice. (Most people will hate you for flipping the switch or just for being involved no matter what you do. The safest bet is to run away if you haven't been seen near the switch yet.)
3roystgnr11y
Interesting - I hadn't heard that added to the problem before. Are you allowed to explicitly consider other psychological effects of your choice? (e.g. presumably people would update their beliefs to account for "marked-as-currently-unused trolley tracks are more dangerous than I thought" and "in-active-use trolley tracks have another level of safety I hadn't considered")
-1Eugine_Nier11y
How is that different from choosing "straight"?
5DanArmak11y
Suppose the switch is stuck halfway between "straight" and "turn". It will take some effort to move it to either position. If you leave it alone, the trolley will take a path at random with equal probability. If you take the time to move the switch to either position, you will be seen. The survivors will thank you and the families of those you chose to kill will sue to put you in prison. Newspapers will print your photo and denigrade philosophers of ethics in deeply misguided editorials. Your unique and memorable name and face will be forever associated with the story. If you don't touch the switch and run away, nobody will know you were there. What do you choose?
-1komponisto11y
Other. The more relevant question is what kind of laws I would enact, rather than what I as an individual would do in the situation.
0thomblake11y
Relevant to what? It's usually brought up in discussion of ethics, not law.
1komponisto11y
If this situation actually arose, there is a substantial possibility that I would not switch the track, in order to avoid being prosecuted for murder. This isn't to say that I "endorse" that (in)action; I would prefer a legal code under which that wouldn't happen, and would vote for legislators who committed to changing the law in that way. Discussions of "law" aren't discussions of which laws to make; those are called discussions of "policy", and they are in my opinion the place where ethical considerations are most relevant.
4RichardHughes11y
Presuming I value the lives of all the people involved equally, I turn on to the side track. If I have a strong reason not to let the person on the side track die - they're a relative, I know them well, they owe me money, I'm in love with them, whatever - I let it go straight. This is a really easy problem if you accept that you're only a marginally good person at best.
2r_claypool11y
Oops, I meant to choose "Accept: turn" instead of "Accept: straight"

Too late, and now they're dead.

6TheOtherDave11y
As, to be fair, are their billions and billions of counterparts in hypothetical-experiment land who are tied to the tracks further from convenient switches.
1A1987dM11y
Is this about what I think would be normatively better to do or about what I predict I would actually do if personally faced with such a situation?
5TheOtherDave11y
Traditionally, trolley problems are asking about normative ethics. My prediction would be "Other: dither ineffectually."
-1Eugine_Nier11y
I voted "lead toward: straight", but thinking about it some more I think the correct answer is: "other, hypothetical isn't sufficiently detailed to determine correct choice".

Newcomb's problem: two boxes or one box?

[pollid:92]

Me: I didn't mean to two-box!

Omega: Why would you share an excuse with an omniscient agent?

Me: Because even if there is no causal connection between me giving an excuse, and me being excused, there may be a logical connection. Also, why would an omniscient agent ask a question?

Omega: Due to meta-level concerns. Obviously.

Abstract objects: nominalism or Platonism?

[pollid:89]

An abstract object is an object that does not correspond to any pattern of matter and energy in space-time. Purported examples of abstract objects are numbers, properties, sets, etc. An object that does correspond to some concentration of matter/energy in space-time is called a concrete object.

Nominalism: Abstract objects do not exist.

Platonism: Abstract objects exist.

Still not sure what this means. Is there some sense in which this distinction pays rent in anticipated experience?

I voted other because of my confusion on this point. I think we need to taboo "exists".

Using my recent attempt at (partially) tabooing "exists" to translate:

Nominalism: We can't rationally care about abstract objects.

Platonism: We can rationally care about abstract objects.

So far Platonism appears to be "winning" according to this definition since UDT is Platonist in this sense, and there isn't really a "nominalist decision theory" that's equivalent or seems as promising.

3DanArmak11y
That just shifts the ground to disagreeing about what is "rational" when arguing about different epistemologies.
1[anonymous]11y
Shifting the ground to an easier, more tractable problem? Awesome.
2drnickbone11y
That seems a rather new argument for Platonism. But what about possibilist versions of Platonism as in "Abstract objects are ones which possibly exist"? It seems quite rational to care about things which might happen, or which might exist without conceding that they actually will happen or actually do exist.
1RichardHughes11y
I voted 'other' to the original question. I would vote 'accept platonism' to this question.
2pragmatist11y
My inclination is to say that it doesn't, and that the disagreement is really just about how to use the word "exist". But there are a couple of ways in which the distinction might have a bearing on anticipated experience. One prominent argument for Platonism is the Quine-Putnam Indispensability Argument, which says that our best strategy for ontological commitment is to believe in the existence of those objects over which our best scientific theories indispensably quantify. So if one cannot dispense with quantification over mathematical objects while maintaining the integrity of our best theories, then we should believe in the existence of those mathematical objects. If one accepts this criterion, then whether or not one believes in the existence of abstract objects depends on whether our best theories indispensably quantify over such objects, so the Platonism vs. nominalism debate depends on science. Hartry Field wrote a book where he tries to axiomatize Newtonian continuum mechanics without any quantification over real numbers. Also, it's plausible that Tegmark's Level IV multiverse hypothesis assumes Platonism, since it requires that mathematical structures have independent existence. So if you believe Tegmark's hypothesis constrains anticipation, then perhaps Platonism does as well.
0Manfred11y
Ah, that sounds a bit like coordinate-free physics.
0DanArmak11y
It's a prerequisite that a question must not pay rent in anticipated experience before it can be part of philosophy.
0diegocaleiro11y
Nope. The prerequisite is that it is uncertain whether it does or not pay rent. See Russell 'Problems of Philosophy" for a characterization of philosophy as "That which may eventually develop into science"...
0A1987dM11y
That reminds me of my aunt saying “philosophy is that thing with which or without which the world would stay the same” (the Italian words for ‘which’ and ‘same’ rhyme).
0DanArmak11y
As far as I can see, none of the questions in this survey have any relation to anticipated experience.
1pragmatist11y
I think a number of discoveries in psychology and neuroscience are relevant to the physicalism vs. anti-physicalism question. I think relativity basically destroys the case for A-theory. The idea of an "objective present" loses all attraction (for me at least) when you realize that there is no such thing as objective simultaneity. I think there's plenty of evidence that God does not exist (and there is plenty of potential evidence that would convince me that He does).
0crazy8811y
There's two things you could mean here. First, you could mean that the notion that their is no objective simultaneity removes your motivation to accept an A-theory. Second, you could mean that it makes an A-theory untenable. I take it you meant the first of these, but if not it might be worth checking out the second half of Craig Borne's book "A Future for Presentism".
-1DanArmak11y
In your examples, evidence from other disciplines has bearing on questions in philosophy. The problem is that information rarely flows the other way. All the philosophical debate on these (real) questions did not contribute significantly to our understanding. And then useful data came in from the relevant sciences and settled them, and would have done so even without the philosophical arguments in place.
1pragmatist11y
Yeah, I was responding to your original claim that none of the questions here have any link to anticipated experience. Your claim here -- that philosophy does not produce any knowledge of use to other disciplines -- is a different criticism, and one that my comment was not intended to address. I think this criticism is also false, by the way. Well, it may be true in the sense that as a matter of fact very few people in other disciplines pay much attention to contemporary philosophy, but it is false that there is nothing of value in philosophy for these other disciplines.
7shminux11y
Another false dichotomy. The word exist means different things to different people.
2komponisto11y
Yes, this is definitely a confused question. "Correspondence" is complicated. People shouldn't use "exists" to mean "corresponds to some pattern of matter and energy" (so apparently I'm a Platonist); yet they also shouldn't ignore the ontological distinction between numbers and atoms (so I guess I'm also kind of a nominalist).
0shminux11y
Life is so much easier for an instrumentalist, for whom "exist" is quite clearly defined.
0TheOtherDave11y
I agree with you that the distinction here gets at what people mean by "exists," but am not sure what makes that a false dichotomy. Personally, I lean towards nominalism precisely because "exist" isn't a verb I would apply to abstract objects... whatever it is that existence consists of, abstract objects don't do that thing.
-3shminux11y
Wikipedia: For example, abstract objects could be considered to exist in the minds of people imagining them, and consequently in some neuronal pattern, which may or may not match between different individuals, but considered to not exist as something independent of the conscious minds imagining them. While this is a version of nominalism, it is not nearly as clear-cut as "abstract objects do not exist".
2siodine11y
That would be conceptualism and is a moderate anti-realist position about universals (if you're a physicalist). Nominalism and Platonism are two poles of a continuum about realism of universals. So, you probably lean towards nominalism if you're a physicalist and conceptualist.
-3shminux11y
I only used the word "exist" in a sentence because TheOtherDave and I agree on the meaning of it, which I doubt that any of the -ists you mention (probably including you) would agree with.
0siodine11y
And what meaning is that?
1[anonymous]11y
That seems to me to be pretty straightforward nominalism to me. I'm having a hard time imagining a more strict nominalist who would call your view an intermediate between his view and platonism.
1TheOtherDave11y
OK, thanks for clarifying. I agree that if I don't have a crisp understanding of what it means for X to exist (such that maybe X implemented as a neuronal pattern exists, and maybe it doesn't, and no amount of data about the world could tell me which it is because I don't know how states of the world map to the existence or nonexistence of X in the first place ) then I can't clearly assert whether X exists or not. For my own part, I'm fairly comfortable refusing to use "exists" to refer to that which abstract objects are doing by virtue of being represented by a particular neuronal pattern, and I'm consequently fairly comfortable identifying as a nominalist (for purposes of this question). If I were instead comfortable using "exists" to refer to that act, I would identify as a Platonist (fpotq). If I was comfortable doing both, or neither, I would choose "Other."
5thomblake11y
Other: Aristotelianism. Abstract objects exist when instantiated. The "form of 2" does not exist in a world of forms somewhere, but 2 billiard balls is an instance of both '2' and 'billiard ball'.
0Rob Bensinger11y
I agree this is an 'Other' doctrine, and 'nominalism' is a bad name for the doctrine of concretism (i.e., everything is spatiotemporal). But isn't Aristotelianism a form of 'nominalism' as defined here? Doesn't your Aristotelianism deny, or refuse to affirm, any "object that does not correspond to any pattern of matter and energy in space-time"? Properties are not ordinarily thought of as objects, and even so if you think that all properties are instantiated, then they do seem to be spatiotemporal, though perhaps in an informational rather than 'matter/energy' sense. Hm.
0thomblake11y
I'm not sure if Aristotle actually believed an object could be an instance of two forms. That's pretty advanced polymorphism.
2[anonymous]11y
In the loose sense of 'form' (cf. Physics I.7), I think objects could: a rabbit has the form of a rabbit, and if it's white it has that form too. That's also consistant with the relationship between primary and secondary substances and non-substances from Categories. On the other hand, in Metaphysics, 'form' is pretty much restricted to the identity of a substance. Still I think Aristotle would be happy enough with polymorphism so long as you made a distinction between substance-forms and non-substance-forms.
0thomblake11y
Sounds about right to me.
1DanArmak11y
What do platonists say the word object means, if some objects are abstract? What is a property that is true of all objects? What are some categories of non-objects?
5pragmatist11y
For most Platonists, an object is something we quantify over. So, for instance, numbers are objects, because we say things like "There is one even prime" (existential quantification), and "All multiples of 6 are also multiples of 2" (universal quantification). Any domain over which we quantify is a domain of objects.
3DanArmak11y
Then what does it mean for an abstract object like 2 to "exist"?
0pragmatist11y
There are many different answers to this question. One common answer is that we should believe an object exists if a theory we accept quantifies over that object. If quantum field theory requires quantification over the integers then 2 (and other integers) exist. Since contemporary physics doesn't quantify over a domain containing phlogiston, phlogiston does not exist.
2DanArmak11y
There's something wrong when smart people argue and disagree over a question when there are many different ideas as to what the words in the question actually mean...
8jimrandomh11y
Other: I hold a Tegmark-style "all mathematical objects exist" view, but additionally hold that this requires promoting the type of the "exists" property from boolean-valued to real-valued, so that this becomes "all mathematical objects have nonzero degree of existence", and most ordinary usages of the word "exists" pick up an implied "as a substructure of" relation to some set of universes. This "as a substructure of" relation excludes platonic objects when present, so nominalism vs. platonism reduces to the word "exists" being a pair of homonyms.
0Rob Bensinger11y
Augustinian!!!!!! hssss
3crazy8811y
I think there's some truth in the claim that this is just a debate over the word "exists" but actually, in philosophy there's a fairly well-regarded meaning of the word and the debate tends to be about this meaning. I think that's one of the reasons comparisons of people from LW with philosophers will be misleading, philosophers use a lot of technical vocabulary that differs from the standard usage. So by "exists" philosophers tend to mean something like "is quantified over in our fundamental theory of the universe". So nominalists in mathematics tend to be arguing something like (and this is a dangerously rough approximate), "we can describe the universe without referring to numbers etc". See Hartry Field for an example. This is rough but actually, platonism begins to seem a lot more plausible when it's understood in the way that philosophers actually use it. I suspect, then, that those from LW and philosophers are just answering different questions here.
2Spinning_Sandwich11y
I think I'm more comfortable with the negation of Platonism than with the positive claim of nominalism, but I suppose in this context we have 'nominalism' = '~Platonism'. Whether what is usually meant by 'nominalism' is the same is as unclear to me as I am uncomfortable with the idea of making a positive claim about it.
2WrongBot11y
Other: Existence is a two-valued function, not one-valued.
1pragmatist11y
Could you elaborate, please?
0WrongBot11y
Sorry for the very belated reply, but I was struggling to find the words to describe exactly what I meant. Luckily, Eliezer has already done most of it for me in his latest post. Thing A exists with respect to Thing B iff Thing A and Thing B are both part of the same causal network. So ArisKatsaris was half-right, but things outside our past and future light cones can be said to exist with respect to us if they have a causal relationship with anything that is inside our past and future light cones.
0ArisKatsaris11y
Wrongbot's idea is probably that something "exists" only in relation to something else. So things beyond the Earth's past and future light-cones don't actually "exist" for us, though they exist for themselves, and similarly things in a mathematical model of a different universe exist in relation to each other, but not in relation to us...
1[anonymous]11y
Other: Abstract objects clearly have some kind of logical existence, but it is very different from concrete existence. Also, I'm confused on this one. Also, the answer doesn't matter, so who cares. Basically Taboo exists
0A1987dM11y
Tree-in-a-forest. I think that "exist" can have (at least) two different meanings and that abstract objects exist in one of those sense but not in the other.

Politics: libertarianism or egalitarianism?

[pollid:87]

Other: utilitarianism

2Manfred11y
Other utilitarianism: symmetrized_Manfred_utilitarianism :P

Other: I think it's a false dichotomy. I think that an ideal system of government will probably sometimes have to sacrifice libertarian principles in favor of egalitarian ones, and sometimes have to sacrifice egalitarian principles in favor of libertarian ones.

4Richard_Kennaway11y
What principles will it use in making such choices?

How happy, safe, productive, etc. people are. I don't see either libertarianism or egalitarianism as terminal values.

0Manfred11y
I believe Richard's point is that e.g. egalitarianism is in fact a system for making these sorts of choices (are people more equal? Yes? Do that!). And of course the principles that you use define your "actual" system, which is neither egalitarian nor libertarian (which would be "are people more free? Yes? Do that!")
-2DanArmak11y
Ideal by what metric? Unless you're a moral realist, there probably isn't such a thing.

Ideal in terms of fulfilling my terminal values, which contain a term for the satisfaction of others.

7DanArmak11y
You're right. My comment was silly and aggressive. (Of course it did not seem that way when I wrote it.) I seem to have a blind spot when I think I see moral realism. My apologies.
8TheOtherDave11y
My real answer is "somewhere in between," but I think that pretty much describes everyone. I ended up answering in terms of current political structures and what direction I think they should move in.
4bramflakes11y
What if I simply don't know?
3RichardHughes11y
I voted 'other' and downvoted the question. Lordy, what the heck are you doin' bringing this in here? D:
3MixedNuts11y
Other: solve scarcity, let everyone have infinite everything.
3AspiringRationalist11y
If libertarian vs. egalitarian is the main axis along which members of a group differ politically, it's a very unusual group. What are we really trying to find out about the group? Possibilities that come to mind include: * The philosophical viewpoints that underlie our political views * Our political preferences along various axes, such as interventionist vs. non-interventionist economic, social and foreign policy * Party preferences
2A1987dM11y
That's the American sense of "libertarianism", right? I take this question to be about the horizontal axis of The Political Compass.
3Jayson_Virissimo11y
For what it's worth, I interpreted it as liberalism versus socialism.
4A1987dM11y
That's the European sense of “liberalism”, right?
0Jayson_Virissimo11y
Affirmative.
1[anonymous]11y
Other: Do the right thing. Consequentialism and all that. Would gladly bow completely to an FAI, but value freedom. Would accept some people doing better than others, but it would suck if the have-nots can't do anything about it. What a simplistic dichotomy...

Justification: externalism or internalism?

[pollid:81]

Other: "Justification" is just another complicated pre-Bayes way of trying to understand what belief is.

0benelliott11y
Same

Externalism: A subject's belief can be justified even if the justification is not consciously available to the subject. For instance, if the belief is formed on the basis of a reliable perceptual faculty, it may be a justified belief even if the subject is not aware that the relevant faculty is reliable or even that the relevant faculty is the source of the belief.

Internalism: A subject's beliefs are justified only if the subject has conscious access to the justification.

Aren't these just different definitions of the word "justified", rather than arguments about what is actually "justified"?

3pragmatist11y
Quite possibly.
2tut11y
Yes. The question is what you mean when you say the word "justified" regarding a belief, without stating a definition.
0diegocaleiro11y
DanArmak, Much of what goes by "philosophy" these days is isomorphic to that, in case that shocks you.

Voted for "externalism", but caring about whether a belief is "justified" is probably a mistake.

0[anonymous]11y
Other: whatever. The correct answer is Bayes, and the engineering approach. Let the philosophers fight over which of these that is.
0TheOtherDave11y
Other: I dithered on this a lot. I sympathize with the meta-answer (as above) of discarding the notion of a "justified belief" in favor of talking about how experiences serve as evidence for beliefs, but it's not clear to me that that precludes engaging with the question at its own level. Yes, whether a belief is "justified" can be expressed more precisely in terms of confidence intervals based on available evidence, but I'm not convinced that it needs to be. I end up saying it depends. I would say I'm justified in believing the two objects I'm looking at are the same size if they look the same size to me, even if I'm not consciously aware of the process whereby I arrive at that belief, even in cases where it turns out that they aren't the same size after all. Which is an externalist position as described here. But I would not say I'm justified in believing any proposition I happen to believe. In some cases I would declare a belief unjustified if I'm not aware of the mechanism whereby I arrive at it. So I'm not comfortable describing myself as an externalist in a broader sense, or even as leaning towards externalism in a broader sense. Thinking about it some more, I suppose this dichotomy dissolves if I'm willing to treat sufficiently vague patterns as a "relevant faculty". "I believe the objects are the same size because that's what my eyes report and my eyes have a good track record about that sort of thing" might qualify as having conscious access to justification, in which case I suppose I'm an internalist... I can't imagine a belief I would call justified for which there isn't some kind of explanation of that sort, however vague, that I can make. But this seems uninteresting.

Analytic-synthetic distinction: yes or no?

[pollid:78]

Yes: There are certain sentences which are true solely by virtue of the meanings of the words involved, so these sentences are not subject to empirical falsification. Example: "All bachelors are unmarried." It is impossible for this sentence to be false, provided the words retain their ordinary meaning.

No: Every sentence is potentially open to empirical falsification. [EDIT: I guess the "No" answer would also be appropriate for those who believe that no sentence is open to empirical falsification, although I would be very surprised if anyone on this site fits that description.]

8benelliott11y
The Yes answer seems obvious, is there some sort of gotcha?
9pragmatist11y
[VOTE BEFORE READING THIS COMMENT TO AVOID PRIMING.] The most prominent critic of the distinction is Quine. You can read about the reasons for his opposition here. A quote:
0diegocaleiro11y
There is also Chalmers 2009 I guess paper about this, which breifly reviews the history of what happened after Quine polarized the topic. Revisability and Conceptual Change. Chalmers attemps (in my view succesfully) to rescue 80% of what matters in the distinction, avoiding Quinean and post Quinean traps.
4pengvado11y
Is "some sentences of first-order logic are tautologies" a sufficient reason to vote yes? If so, clearly we should be talking about that rather than complicated human-language examples like bachelors. If not, what's the difference?
2torekp11y
The answer to your first question will be controverted for pretty much the same reasons that the analytic-synthetic distinction itself is, I think. Quineans will claim that insofar as those sentences of first order logic are actually used in science, they become enmeshed in the holism of cognitive meaning. Good point to raise, nevertheless.
4Manfred11y
Other: Unique meanings are a useful approximation, but if you stretch an approximation thin enough it develops holes. For example, "bachelor" in middle english refers to a squire, and squires can be married.
2Spinning_Sandwich11y
I probably should have voted for "Other," but I voted for "Lean toward: yes" because I still outright agree in certain contexts. Quine's Two Dogmas is certainly enough to make me doubt the usefulness of the analytic/synthetic distinction as regards ordinary language, but for formal languages, this is not the case. It's also not clear to me whether it's impossible to construct a language (for communication) clear enough to make sense of analytic/synthetic distinctions. This is one of those wonderfully agnostic positions that philosophy often leaves me with.
2[anonymous]11y
Other: Lean toward two-dimensionalism.
2Alicorn11y
Can you elaborate on this? I haven't heard of it in this context before.
3[anonymous]11y
The idea is that the meaning of an utterance isn't just one thing; it's kind of the overlap between two distinct propositions: the sense (that is, the concept or idea by which we find the referent), and the referent, the actual entity to which it refers. The standard textbook example is the word "water"; the sense of "water" would basically correspond to a descriptive, conceptually encoded "water"-iness, and the referent would be the substance itself. Basically, you've got your ideas about water, you've got the abstract entity you recognize or impute as a member of the reference class (the substance H2O considered in abstract, or a pond, or a glass of clear odorless transparent liquid on the table in front of you), and both are relevant to determining the actual semantic content.
2[anonymous]11y
This was the most surprising poll result, to me.
0jsteinhardt11y
I assume you also expected far more of the (currently) less popular answer?
0Manfred11y
Huh, yeah, I'm pretty surprised too. Possibly even for the same reason :P
-1TheOtherDave11y
Huh. Well, I'm willing to be convinced. So, OK, sticking with pragmatist's example, can you summarize the conditions under which "All bachelors are unmarried" becomes false while the words retain their ordinary meaning? (I recognize that we might just turn out to disagree on what their ordinary meaning is, which I think would be uninteresting, but I'm hoping it won't come to that.)
2Alejandro111y
According to Quine's meaning holism, explained by pragmatist here, the concepts of "bachelor" and "marriage" are embedded in a wider network of concepts like "human", society", "legal relation", etc, and their use presupposes an amount of "truisms" such like that there exists humans, that humans can get involved in socially-endorsed legal relations, etc. I find it conceivable that some of this truisms turn out to be false (e.g. imagine you are a brain in a vat) and that the entities you think of as humans are better described with a vastly different network of concepts, inexpressible with our currently existent ones. It may be that after you become aware of this and you acquire the better set of concepts, you will find your old concepts of "bachelor" and "married" confused to several degrees, and in a way such that the best way to make them survive implies that not all bachelors are unmarried. One motivating example often used by Quineans is the law of excluded middle ("all meaningful propositions are either true or false"). It might seem analytic based on the meaning of "proposition", "true" and "false", but there are interpretations of quantum mechanics in which it is false. Whether those interpretations are the best ones is beside the point; the point is that there are several philosophers of physics who find the negation of the law of excluded middle conceivable in a new conceptual structure more appropriate to describe the new facts of quantum mechanics. The Quinean claim is that all propositions are revisable in this way.
3TheOtherDave11y
Sure, I can believe that. I mean, I can't imagine it, but I believe it's possible in some way that I can't imagine. I certainly agree that meaning is holistic in the sense pragmatist explains, such that the meaning of such a sentence can change based on systemic effects. But to believe that, and believe that "bachelor" and "marriage" have their ordinary meanings at the same time, is beyond me.
8Alejandro111y
The contention is that "the ordinary meaning" of a word is a fuzzy and ill-defined concept, once holism creeps in. Consider another example (used by Putnam, I think). Physicists first introduced the concept of "momentum" as the product of mass and velocity. A central fact that made the concept useful was that momentum is conserved in an isolated system. Later, with relativity, it became clear that the conserved quantity is not really the product of mass and velocity, but includes a speed-of-light dependent factor as well. Physicists started then calling this quantity "momentum". Now, was this a change in the meaning of the word "momentum", or a new fact discovered about the same physical entity momentum? This would seem to depend on whether the first early modern physicist who used the word intended "momentum" to have a fixed meaning as the product of mass and velocity, or as the quantity conserved in an isolated system. But he probably didn't make his intention clear, and in any case his private intention does not matter if meaning is social and holistic. Even if most pre-Einstein physicists would have (if questioned, which they weren't) agreed that "momentum" meant definitionally mass times velocity, post-Einstein physicists may be perfectly justified in saying they would have been wrong, that Einstein made a new physical discovery about the same quantity they were trying to talk about and not merely changed the meaning of words. What is "the ordinary meaning" (pre-Einstein) of "momentum" is not a question with a well-defined answer; the relevant unit of meaning was the whole physical theory, which was replaced by a new one, and we cannot make a clean distinction between which were changes in meaning and which were changes in factual beliefs. It is more difficult to imagine something like this happening for "bachelor", but according to Quineans, the difference is only of degree.
5fubarobfusco11y
Considering the present controversy over "the definition of marriage", I think we can imagine many such cases. Is a man who has lived with the same woman for ten years — but has never had a wedding — a "bachelor"? How about a man who has had a commitment ceremony with another man? (Does it matter if the invitations said "marriage" or "commitment ceremony"?) A man who has a marriage of convenience to a woman he has never slept with, for purposes of immigration? A man from a culture where he was, as a young boy, married by his family to a young girl, but who has left that setting and never seen her since? A man who believes he is married to a particular woman, but subsequent careful inspection of family history reveals that she is his long-lost sister and thus the marriage is invalid for incest? A Catholic priest? Just as subsequent physics discoveries can problematize the definition of "momentum", subsequent social and personal-history discoveries can problematize the definition of "bachelor". The "ordinary meaning" is only had by choosing to ignore problems.
2TheOtherDave11y
OK, I think I follow. For my own part, I would say that if we take the relevant unit of meaning to be the whole physical theory (a position I find compelling in principle, if unwieldy in practice), it follows that changing the physical theory does not preserve preexisting meanings. I would not say that the meaning of "momentum" changed, precisely, but that "momentum" acquired a new meaning in addition to its old one, and anyone talking about momentum in a relativistic context is using the new meaning, even though people talking about momentum in a non-relativistic context can go on using the old meaning. (I would also say that the intent of the first physicist to use the term is effectively irrelevant.) This also implies that people who try to copy over assertions about momentum from non-relativistic contexts to relativistic ones are essentially confusing homophones... similar in principle to what happens if I try to copy over assertions about monarchs from lepidopterological contexts to governmental ones. But, OK, I can understand how someone could sensibly argue that no, the meaning is preserved, because the meaning was always fuzzy in the first place, we just became aware of the fuzziness late in the game. (This seems to in turn depend on a strongly externalist account of meaning.)
1[anonymous]11y
Not the source of my surprise. Thanks for priming everyone who reads this thread before voting, though.
0J_Taylor11y
If you are interested in exploring this issue, Quine's Two Dogmas is probably the best place to start. http://www.ditext.com/quine/quine.html
0arundelo11y
Other: I think the distinction clearly exists (in the sense that it's a human concept that carves reality more or less at the joints, so that a person can learn the definition and then come up with examples), but it's not useful for much apart from arguments-about-words-rather-than-things, and, because of the fuzziness of definitions, in practice it's more like a continuum than a strictly binary distinction.
0[anonymous]11y
Wow. I voted "No" because modus ponens and such are just procedures that produce evidence, not some kind of magical truth juice. I guess if "Yes" means you can construct consistent definitions that are not empirically verifiable, then I'd vote Yes.
0Spinning_Sandwich11y
Unless you buy into Kant's synthetic a priori arguments, that's really all analytic means. Of course, in practice it's far more interesting & complicated, and it even leads to the kind of applications that have made secure internet commerce possible, not to mention the computers we use to do that. At least, on some days I think that's what 'analytic' means. Maybe.
0[anonymous]11y
I'm not interested in this philosophy stuff, but our names have an amusing symmetry. internet fist bump
0TimS11y
Crud. Misread the poll as about the viability of the a prior / a posteriori distinction. (True without empirical content {the usual example is first-order logic or mathematics} vs. truth-value only after empirical content). History note: Kant wanted to assert the existence of a priori synthentic statements - a position I find nonsensical.

Mental content: externalism or internalism?

[pollid:76]

Externalism: The representational content of our mental states (e.g. what objects our beliefs are about) is dependent upon properties of our external environment, not just upon properties of our brain state.

Internalism: The representational content of our mental states is fixed by our brain state.

This looks like an unheard falling tree problem, the problematic term being "the representational content of our mental states".

1[anonymous]11y
Voted other for essentially this reason.

Agreed with Richard above, it's hard to know what to do with "the representational content of our mental states". How would I know if the representational content of one of my mental states had changed? What would I expect to observe differently?

That said, I voted "internalism", roughly on the grounds that while I can posit things that might deserve the label "an aspect of the representational content of a mental state that depends on properties of my external environment," I don't actually seem to care about any of them.

4novalis11y
If you take Dennett's intentional stance towards our mental states, I think you can only do so by considering the external environment that the brain is operating in. Which I guess makes me an externalist (which is what I answered), but I'm not sure whether that's the traditional definition.
0drnickbone11y
Also voted for "externalism" for the same reason.
0diegocaleiro11y
I wrote a book on Dennett, read the vast majority of what he wrote, and agree with both of you on the Intentional Stance --> Externalism. If that is worth something. Dennett's notional worlds are not externalist, but not contain properties philosophy usually ascribes the category of mental states, such as beliefs. "Beyond Belief" for more on Dennett on that.
2Oscar_Cunningham11y
I voted "Lean toward: externalism", but the I get the feeling that even asking the question shows that you're barking up the wrong tree.
0Swimmy11y
Voted "other" for this reason. Seems like a wrong question.
1faul_sname11y
Other: What? I'm not understanding the question.
2pragmatist11y
Does this help?
0faul_sname11y
Yes. I presume they haven't had time to notice they're in different physical environments (as they will diverge in a matter of milliseconds as they see different things and different neurons fire).
1[anonymous]11y
Other: embodied cognition (externalism but moreso).
0[anonymous]11y
Other: What? The brain state simply is. You can give it meaning, but meaning is a two place function. I'm probably confused.
2pragmatist11y
Think of it this way. There are two people, A and B, who are exact physical duplicates of one another. They are, however, embedded in different environments. If you think that A and B must necessarily have all the same beliefs, you are an internalist. If you think their beliefs can be different, you are an externalist.
0[anonymous]11y
They are the same, they pretty clearly have the same beliefs. I infer from the disagreement that this is about "knowledge" (which is a lot more confused than belief). With knowledge, it depends how you define it (does it have to be true?). Other.
2pragmatist11y
No, it is not about knowledge, it's about belief (or any other representational state). An externalist believes that the content of a belief -- what a belief is about -- is determined by causal entanglements with the environment. I have beliefs about Eliezer Yudkowsky because my brain states are causally entangled with Eliezer Yudkowsky. These beliefs need not qualify as knowledge. Say a physical duplicate of me appeared on Mars by a random thermal fluctuation. Even though that duplicate has the same brain state as I do, his brain state is not causally entangled with Eliezer, so the externalist would say that he does not have beliefs about Eliezer. If you think this is obviously false, you are most likely an internalist.
0[anonymous]11y
keyword "about".
0asparisi11y
Other: While the only things I can directly experience are my brain states, my senses allow me to have indirect content regarding my external environment. While every belief I have regarding external objects is about some brain state, I only have the representational content because it is important for me to match my mental state to the external-world. There is a relationship between the external-world state and the mental representation that cannot be ignored.

Mind: anti-physicalism or physicalism?

[pollid:75]

Physicalism: A physical duplicate of our world (i.e. a world in which all the same physical properties are instantiated at the same space-time locations) must necessarily also be a mental duplicate (i.e. all mental states instantiated in that world must be identical to the mental states instantiated in this one).

Anti-physicalism: The denial of physicalism.

2Vladimir_Nesov11y
In what sense is a duplicate distinct from an original in these definitions?
1prase11y
"Same space-time location" means "same relative distances and time intervals within each world"?
0pragmatist11y
Yes, that is what it means.
0BrassLion11y
Is this the same as asking whether there is a 1:1 correspondence between mind states and brain states?
0Jayson_Virissimo11y
"Other": I maintain a physicalist stance (à la van Fraassen), but do not affirm physicalism.
0[anonymous]11y
Oops I accidentally antiphysicalism. put me down for the correct answer (physicalism).

Teletransporter (new matter): survival or death?

[pollid:107]

Teletransporter: You are placed in a machine that will instantaneously disintegrate your body, in the process recording its exact atomic configuration. This information is then beamed to another machine far away, and in that machine new matter is used to construct a body with the same configuration as yours. Would you consider yourself to have survived the process, and teleported from one machine to the other ("survival")? Or do you think you have died, and the duplicate in the far away machine is a different person ("death")?

4torekp11y
It's important to distinguish between and In other words, it's important to avoid the worst argument in the world. I believe that the former statement is implied by a uniquely best answer to a set of verbal questions - but at the same time, it's a mere technicality.
0BrassLion11y
Voted Lean: Death but want to change my answer to "have no bloody clue". For the record, when I first thought about this I was accept: death.
1kybernetikos11y
I tend to think death, but then I'm not sure that we genuinely survive from one second to another. I don't have a good way to meaningfully define the kind of continuity that most people intuitively think we have and so I conclude that it could easily just be an illusion.

Knowledge claims: contextualism, relativism, or invariantism?

[pollid:103]

Contextualism: The truth of a knowledge claim depends on the context in which it is uttered. A claim such as "Alice knows that she is not in the Matrix" might be true in certain contexts (when explaining to someone in ordinary conversation why Alice didn't lose sleep over the movie Matrix) but false in other contexts (when uttered in an epistemology class in a discussion about the possibility of us being in the Matrix). The usual analysis is that the same sentence about knowledge expresses different propositions in different contexts (just like the sentence "It's raining here" expresses different propositions in different contexts).

Relativism: Whether a subject possesses knowledge of a certain proposition is relative to a set of epistemic standards. Relative to one such set, she might know that the proposition is true, while relative to another set, she does not qualify as knowing this. So, strictly speaking, "knowledge" is a three-place function, taking as arguments a subject, a proposition and a set of standards.

Invariantism: Knowledge claims are either true or false simpliciter. Their truth does not vary depending on context, and they are not relativized to epistemic standards.

EDIT: A couple of people have said that the difference between contextualism and relativism is unclear. I have tried to clarify in this comment.

0thomblake11y
That doesn't match my intuition about what "relativism" means here, but I haven't taken any epistemology in a while so I'm more likely to be wrong, I think.
0pragmatist11y
What is your intuition?
0komponisto11y
Don't see much difference between contextualism and relativism.
3pragmatist11y
In relativism, a single utterance of a knowledge claim can be assessed differently depending on the assessor's epistemic standards. So the truth value of a knowledge claim can vary depending on who is doing the assessing. In contextualism, the truth value of a knowledge claim only varies if the context of utterance is different. A single utterance of a knowledge claim will have a fixed truth value, independent of who is assessing the claim. You only get variance when you vary the context in which the utterance is made, just like you only get variance in the meaning of "It's raining here" if you vary the context in which it is uttered. Does that help?
4TheOtherDave11y
Er... well, not yet, but might be a step towards helping. I get that, if the conditions "different assessor" and "different context of utterance" are separable, then relativism makes distinct claims from contextualism. That is, if I have a different assessor but the same context (or even the same assessor whose epistemic standards have changed, and the same context), then contextualism asserts that the truth value of my claims is necessarily unchanged, and relativism asserts that it might change. What I can't fathom is how that happens, even in principle. Isn't the assessor, and the assessor's epistemic standards, part of the context of the utterance?
4[anonymous]11y
So imagine you read an ancient text in which the claim "the voice of Zeus follows upon his blows" appears as a description of the relation between thunder and lightening. THe assessor and facts about the assessor are nowhere part of the context of the utterance, because the utterance was made thousands of years before you were born. An invariantist would say 'that's false, there's no such thing as Zeus'. A relativist would say 'That's false to me (no Zeus), but it might have been true for the person who wrote it if they had different standards for truth." A contextualist would say 'So Zeus doesn't exist, but the voice and blows of Zeus are just terms for lightening and thunder. If someone made this comment knowing what we know about stoms, it would be false. But I assess this statement as true, because I take the utterer to be talking about lightening and thunder.'
2TheOtherDave11y
(nods) I think I get it now... thanks! (More thoughts here)
3pragmatist11y
The assessor doesn't have to be a person standing there listening to the utterance. Suppose I told you that my friend Alice says that she knows OJ Simpson killed his wife, and I ask you what you think about her knowledge claim. If you were a relativist, you would evaluate the truth of the knowledge claim in accord with your epistemic standards, or you might even say, "Well, relative to standards X her knowledge claim is true, but relative to standards Y it's false." If you were a contextualist, on the other hand, you'd ask me "In what context did she make this knowledge claim?" and then base your evaluation of the knowledge claim on my answer. You allow the context to set the epistemic standard you use for evaluation. Another example: Suppose Bob says "Alice knows P" in one context, and Charlie says "Alice doesn't know P" in another context. For a contextualist, it might be the case that Bob and Charlie are not disagreeing at all. If the contexts are sufficiently different, you can't pit their knowledge claims against one another. A relativist, on the other hand, can pit the knowledge claims against one another, by relativizing them to the same epistemic standard. Only one of them will be true according to that standard.
2TheOtherDave11y
Ah! (lightbulb goes on) Throughout, I have been implicitly understanding "context" to mean context of evaluation. Which is not what we mean at all, we mean context of utterance. Which, indeed, you even said explicitly, and I failed to read carefully enough. Yes, this makes perfect sense. Thinking about this now, I think I endorse contextualism, even though attempting to implement it gives me a headache. That is, whether I'm comfortable saying that you actually know X is a function of what evidence for and against X I believe you're aware of, but my brain strongly tends to replace (my beliefs about) what evidence you're aware of with what evidence I'm aware of. I no longer remember what my vote was. Thanks for your patience.
0TheOtherDave11y
I'm not really sure why the distinction between contextualism and relativism makes a difference.
0pragmatist11y
See here.
0thomblake11y
Basically, philosophers are sufficiently agreed on epistemology that both mean basically the same thing for the lay person's understanding. There's even a species of invariantism called "subject-sensitive invariantism" that is indistinguishable from contextualism to the uninitiated.
0Rob Bensinger11y
None of the prior discussion reflects an understanding of 'contextualism' as standardly conceived by philosophers (if the Stanford Encyclopedia exposition is representative of philosophers' views). So I suspect the polling data for this question will need to be tossed out. Here's a clearer explanation of the difference between these doctrines: contextualism = The semantic thesis that 'x knows y' may vary in truth-value depending on the social and psychological status of the knowledge-attributor. I.e., 'x knows y' often or always fails to have a determinate truth-value, unless it is clear from context that we are really saying 'x knows y relative to evaluator z,' where z is someone evaluating 'does x know y?' Thus, a better name for 'contextualism' would be 'attributor contextualism' (which it has indeed been called). Note that contextualism does not imply that the distribution of knowledge in the world is arbitrary or just a matter of subjective opinion; there may be very strict constraints on what sorts of 'subjective opinions' held by an evaluator affect knowledge-relative-to-an-evaluator. For instance, it is plausible that 'I know I have hands' would count as true if the evaluator were your psychiatrist, but would count as false if the evaluator were someone with whom you were debating the Simulation Hypothesis. That's not because of the evaluator's mere opinions; it's because there are higher standards for knowledge in metaphysical debates than in everyday conversation. An evaluator with crazy, unrealistic standards wouldn't have his/her own, equally legitimate beliefs about what counts as knowledge; s/he would just be consistently in error. Nor is contextualism a meta-semantic claim about how the word 'knowledge' varies across linguistic communities; rather, it is the semantic claim that 'knowledge' in all (standard-English-speaking) contexts would frequently be judged to vary based on the state of the evaluator. Contextualism could turn out to be false for
0pragmatist11y
In your discussion of contextualism, you are conflating "evaluator" and "attributor", I think. An attributor is someone who makes a knowledge-claim, i.e. attributes knowledge of some proposition to someone (including, possibly, to himself). An evaluator is someone who judges the truth of the knowledge-claim made by the attributor. So if you say "I know I have hands" to a psychologist, you are the attributor, not the psychologist. You are the one making the knowledge-claim (about yourself, in this case). The psychologist is the evaluator, and according to contextualism she will (or possibly should) evaluate your claim according to your context, not her own. So if the psychologist hears about you making the claim in the context of a discussion of the Simulation argument, she should probably judge it false, irrespective of the context in which she is situated at the time she is making the evaluation. I believe this agrees more or less with the definition (and discussion) above. Your definitions of relativism and invariance seem to agree with mine.
0Rob Bensinger11y
I wasn't aware that there was an established distinction between the two. Thanks for the information! Though nothing of great weight can rest on it, since: 1. Every attributor is a (self-)evaluator. Asserting 'p' is equivalent to asserting 'p is true'. 2. Every evaluator is an attributor. To determine that some attributor's knowledge-claim is true or false, one must oneself attribute knowledge (or lack thereof) to the relevant agent. Yes. But if the psychologist evaluates your claim, by weighing in on its truth or falsehood, then the psychologist becomes a distinct attributor. I varied the psychologist or metaphysician as attributor, rather than paying much mind to self-attribution/self-evaluation, simply because I thought it would be less intuitive to talk about a person's knowledge-claims failing to meet his or her own psychological/social state. But, sure, strictly speaking I could have just varied the psychological state of someone self-attributing knowledge, and thereby made that person's own beliefs about his/her knowledge true or false. (At least, I think standard contextualist theories allow this.) Interesting. I think it's more complicated than that. For instance, I think contextualism predicts that the psychologist, in a later dinner conversation with a metaphysician friend, might say: 'Earlier I told a patient that he knew he had hands; but, of course, really he doesn't have hands.' The contextualist interprets this as meaning that the psychologist's state has importantly changed, hence her knowledge-attributions have changed, hence her knowledge-evaluations have changed. (Presumably part of the reason the psychologist's knowledge-attributions have changed in this case is that she's in a social context that includes a metaphysician with psychologically embedded 'higher standards'. I.e., contextualism predicts that social overlap produces synchronizations in correct knowledge attribution.) Note that an invariantist might interpret the same data very
0drnickbone11y
Lean towards contextualism, but with some problems. For instance: "I know that I have hands" "I don't know that I'm not a brain in a vat" However, I've just asserted them side by side in the same context (of a Less Wrong discussion). And I don't think relativism helps much either: can the assessor change their epistemic standards so quickly in the space of two consecutive sentences??
2Kindly11y
You're allowed to be mistaken or lying. I suspect a contextualist would argue that exactly one of your assertions is true, depending on what the context of the discussion is.

Aesthetic value: objective or subjective?

[pollid:101]

6thomblake11y
This is one of those cases where I'm not sure exactly what "objective" and "subjective" are supposed to mean. Probably 2-place words, but probably objective ones.
3Vaniver11y
I went with Other because I think aesthetic judgments are mostly the same for humans, but will be whatever evolution spits out for non-humans. There's some objective (aesthetic value is a product of evolution) and some subjective (because it's a product of evolution, it's environment-dependent and subject to variation). For all I know, though, that position is Accept:subjective, since 2-place words would be a radical new insight to most philosophers who pick Accept:objective.
2hankx778711y
Of course this question is universally (snerk) misunderstood as "objective" = "universal", which are not actually synonymous.
2pragmatist11y
Other: I'm genuinely undecided about this. I don't think I lean substantially in either direction.
1komponisto11y
Other: a complex weighted mixture of both, and varying according to context. Similar to ethical value.
-2RichardHughes11y
I'm not sure how anyone could argue that aesthetic value is objective when humans regularly disagree about the aesthetic value of things. It's a pretty stern counterexample.
8DanArmak11y
Humans regularly disagree about lots of objective things, because they're wrong about them.

Meta-poll: this is not one of the original poll questions. It's just something I wanted to ask.

What is your opinion of modern philosophy, if the questions in this survey are taken as representative, important, unresolved issues in the field?

Interesting questions: most open philosophical problems are meaningful, useful, or interesting, and it is worthwhile to research them. If philosophers come to a broad agreement on a currently open issue, non-philosophers should pay attention.

Interesting debate: most philosophical problems are confused debates, e.g. over... (read more)

8TheOtherDave11y
Other: I often find these sorts of questions useful as a way of clarifying my own understanding of related subjects, and I think clarifying understanding can lead to pragmatic value even in the absence of an agreed-upon answer. That is, sometimes it is useful to go from "I am confused about X" to "there are three possibilities (X1, X2, X3) and I know what each one entails but I don't know how to choose among them", even though the question remains equally unanswered. This is similar to your "interesting debate" option, I suppose, but different enough that I felt uncomfortable picking it.
3Swimmy11y
Philosophical problems as a whole are a mix of all 3, and I don't know enough about modern philosophy to empirically determine which answer reigns in the "most." Voted "Other."
3Oscar_Cunningham11y
"Uninteresting", but perhaps only due to Sturgeon's law.
1diegocaleiro11y
There probably is a gigantic bias to the "Uninteresting" amount of responders. If you find those uninteresting, you wouldn't get here in the first place. So, given now it is about 25% "Uninteresting" I'd guess more than 50% LWers are of that opinion.
1magfrump11y
Other: unanswered philosophical questions are about evenly distributed between interesting questions that will soon be matters of engineering, confused and revealing questions, and historical nonsense. I produced easy examples of all three categories without trying.
1Vaniver11y
This book makes the argument that (paraphrased, and put into LW terms) most philosophy is uninteresting because their curiosity doesn't seek to annihilate itself. Instead of asking "how can we actually improve our knowledge?" they bicker over the definition of JTB. The tools and insights of philosophy can be useful when you try to answer the practical questions, but most controversial topics are controversial because there are a lot of wrong ideas there, not because it's a hot new empirical question (Higgs: does it exist? If so, how big is it?).
1TimS11y
Uninteresting artifacts of history. But some (moral realism or not) are vitally important to figuring out effective social engineering. But I'm in a minority in this community in thinking that social engineering is desirable (other than as an inevitable effect of physical engineering/ technological progress).
1Manfred11y
ADBOC with talking about "most," so I voted other. But maybe I should have voted "Uninteresting."
1Alejandro111y
I voted for "Interesting questions", because a slight majority of the polled questions fall in that category to me, and that matches the literal meaning of "most". But when 30-40% of the key questions of a discipline look "Uninteristing", it is not a great endorsement for it.
0thomblake11y
Other: Philosophical problems primarily serve to encourage creative thinking and asking more abstract questions about the world. Philosophical debates primarily serve to remind us that intelligent, well-meaning people can disagree about pretty fundamental things, so we should tread carefully when assuming our "opponents" are idiots or monsters.
0Spinning_Sandwich11y
This depends a great deal on both which branch of philosophy we're talking about & who is evaluating that particular branch's usefulness. For example, I find developments in logics, philosophy of science, & general epistemology to be of great interest, and I perceive all three topics to be advancing (listed in order of priority as that goes) as the years go by. I'm sure others feel differently. It would be hard to get past the fact that, especially between the different branches of philosophy, there is a great deal of "philosophy of language" that is or must be done just to get at what anyone's talking about. But that is, to some extent, true of any field with a technical language. So I guess all four answers make sense in some sense.

Language: Russellianism or Fregeanism?

[pollid:77]

Russellianism: The meanings of our (referential) words are the objects to which they refer. When I say "Socrates is mortal", the meaning of the word `Socrates' in that sentence is a particular person who lived in ancient Greece.

Fregeanism: The meanings of our words are not directly objects in the world but the particular way we conceive of those objects. Two words referring to the same object can have different meaning since they correspond to different ways of conceiving the object. For instance, "morning star" and "evening star" both refer to to the same object (Venus), but they have different meanings.

0[anonymous]11y
What exactly do they mean by "meanings"? Do they mean "mental state triggered by the word" or "what the word is referencing"? Because it could go either way, depending on which definition of "meaning" we're using.

Other: Seems like a semantic problem about the word "meaning".

4Wei Dai11y
Not really. We can frame the debate between Russellianism and Fregeanism in pragmatic terms: is it useful to model expressions as having or relating to Fregean "senses" (ways of thinking about, objects, properties, and relations) in addition to "intensions" and "extensions"? Note that philosophers of language are already quite aware of the need to avoid purely semantic debates about the word "meaning". As evidence, see this paragraph from the SEP:
7pragmatist11y
Both Russellianism and Fregeanism make assumptions about the way language is related to the world that I reject.
5[anonymous]11y
What assumptions do you have in mind?
1novalis11y
Other: This.
0[anonymous]11y
Likewise.
0diegocaleiro11y
I take both views to be inconsistent with cognitive neuroscience. Or, at a more abstract, simplified, level, with Yudkwosky's suggestion of how to think about conceptual clusters, http://lesswrong.com/lw/nl/the_cluster_structure_of_thingspace/ There is also an important distinction between "Language" and "Languages" made by Lewis, relating to how people think about language as within, or separate from the world. I'm unable to find source though.
4bramflakes11y
I'm not sure I entirely understand the question. Isn't it just the distinction between connotation and denotation?
3Manfred11y
Other: Yes.
1faul_sname11y
What exactly do they mean by "meanings"? Do they mean "mental state triggered by the word" or "what the word is referencing"? Because it could go either way, depending on which definition of "meaning" we're using.
0[anonymous]11y
I think that makes you a Fregeian.
0komponisto11y
Really? It seems to me just the opposite: that a Fregean believes the two concepts are the same.
3[anonymous]11y
Well, Frege's big thing (the big thing that didn't fall over, anyway) is a distinction between 'sense' and 'reference', where the 'sense' of a word is something like what we mean by it, and the reference of a word is the actual, real thing the word is about. He came up with this to explain why someone could know the meaning (in the sense of 'sense') of 'the evening star' and 'the morning star' without knowing that they're in fact the same thing (they have the same referent, i.e. Venus). See: http://en.wikipedia.org/wiki/Sense_and_reference
0komponisto11y
Yes, but I was going by the definition given above. No claim that Frege himself was a "Fregean"!
1TheOtherDave11y
Voted Other. I would say the meanings of our words are the desired state changes in the world correlated with the use of those words. I don't know if that position has a name.
0amcknight11y
It seems to me that we can mean things in both ways once we are aware of the distinction.
-2[anonymous]11y
Other: Fregean thing seems right, but talking about the "meaning" of words is stupid. Words are symbols that cause the reader/listener to construct certain thoughts. Useful for communication, I hear.
-4Eugine_Nier11y
Other: some words are Fregeanian others are Russellian.

Proper names: Fregean or Millian?

[pollid:106]

This is the same distinction as Russellianism vs. Fregeanism, except applied specifically to proper names. I think in the Philpapers survey, this question replaced the Russellianism vs. Fregeanism one.

Fregean: The meaning of a proper name is a way of conceiving of its bearer. Different names for the same bearer may be associated with different ways of conceiving, and thus have different meanings. For instance, "Superman" and "Clark Kent" have different meanings.

Millian: The meaning of a proper name is its bearer. The meanings of "Superman" and "Clark Kent" are identical.

8[anonymous]11y
Other: both can be true depending on the situation. This can be only meaningfully interpreted as a question of psychology (what goes on in people's heads), there's no way in which one method of ascribing meaning to names is 'truer' than the other so asking to choose between the two looks like a confused question.

Logic: classical or non-classical?

[pollid:104]

Classical: The standard kinds of logic that you learn in undergraduate logic classes are the best (or right) logics, the ones that best model (ETA: idealized versions of) our inferential processes. Examples of classical logics are Boolean logic and first-order predicate calculus. Classical logics are bivalent (sentences can only be true or false), obey the principle of the excluded middle (if a proposition is not true, its negation must be true) and obey the law of non-contradiction (a proposition and its negation cannot both be true).

Non-classical: The best logic is not classical. Non-classical logics usually reject the principle of the excluded middle or the law of non-contradiction. An example of a non-classical logic is dialetheism, according to which there are true contradictions (i.e. some sentences of the form "A and not A" are true). Proponents of non-classical logics argue that many of our scientific theories, if you probe deeply, involve inconsistencies, yet we don't regard them as trivially false. So they claim that we need to revise the way we understand logic to accurately model our inferential processes.

6komponisto11y
Is that the right criterion? Or should it be: the ones that best model the correct inferential processes, whether or not we humans adhere to them?
0pragmatist11y
Good point. I've edited to reflect this.
1TheOtherDave11y
I lean towards classical, but with the proviso that we have to be careful about what counts as a statement. Sneak in a statement with ambiguous truth values, and classical logic halts and catches fire. Personally I'm OK with rejecting such statemetns.
0A1987dM11y
What does Bayesian probability theory count as?
3Jayson_Virissimo11y
Bayesian probability is an extension of classical logic. I don't think philosophers consider it to be non-classical.
0kilobug11y
In my AI lessons, the "non-classical logic" course including all the probabilistic theories : fuzzy logic, Bayesian, ... that's why I voted "lean : non-classical", but I guess it's just a matter of vocabulary.
0A1987dM11y
Okay, so “Accept: classical” be it.
6pragmatist11y
Other: Different logics are appropriate for modeling how one should infer in different domains. Classical logics are fine for many applications but it is possible (maybe even plausible) that non-classical logics will be better models for certain applications. For instance, fuzzy logic (a many-valued logic) has been successfully employed to control subway systems and build thermostats.
3Richard_Kennaway11y
And yet the metalanguage is always classical logic. Even the most enthusiastic proponents of other systems never use them to talk about those systems. So I go firmly with "classical".
3pragmatist11y
That seems consistent with my view. For the specific application you mention -- talking about logical systems -- classical logics are our best models. It could still be the case that other logics are better for other applications. What makes this particular application the trump card, so that the fact that classical logic is best for doing metalogic means that it is the best simpliciter?
3Richard_Kennaway11y
First, I shall ask the question "what is logic?" And I shall answer it. In the context of the present poll, "logic" means those methods of reasoning that are guaranteed to produce, from true premises, only true conclusions. And the poll is asking whether classical logic is it. Particular formalisms used to model particular things are not, in this sense, logic, although they may be expressed in logic. For example, number theory is not logic. Neither is geometry, or physics, or probability theory. Neither, I claim, is fuzzy logic, despite the word "logic" in its name. You can say, "here is a set of functions (which I shall call fuzzy logic truth tables), and here are some theorems about how they behave (which I shall call fuzzy reasoning), and here are some physical systems whose description uses these functions." That does not mean that those functions are actually a form of logic, as I just defined it. Bang-bang controllers like the room thermostat were invented (in 1883) long before fuzzy control theory (about which I've heard anecdotally that the term was invented only to avoid someone's patent claims). The closest anyone has come to promulgating an alternative system is intuitionistic logic, which is a pessimistic version of classical logic, in which the axiom of the excluded middle is dropped. In intuitionistic logic, you cannot infer P from not-not-P, or carry out proof by contradiction. However, I think intuitionism is simply a mistake, a historical accident which would never have happened if there had not been a half century between the codification of mathematical logic and the invention of the computer. Everything that is useful in intuitionism is given by computability theory and classical logic.
4pragmatist11y
I agree with you that a logic is an account of truth-preserving inference. But, by this definition, fuzzy logic absolutely qualifies as a logic. The rules of inference in fuzzy logic are truth-preserving, provided we're talking about "full" truth, i.e. we're not in the realm of fuzziness. There are other non-classical logics, besides intuitionism, that also provide accounts of valid inference that are truth-preserving. Relevance logic, for example.
0Richard_Kennaway11y
I still see those as mathematics, rather than logic, and the same goes for all other non-classical systems, such as all the modal logics. All of these are more like group theory than they are like logic, in the fundamentalist sense of "logic" I read the poll as talking about. They axiomatise certain mathematical objects, but not the general process of valid reasoning itself. That, I claim, is a problem completely solved by the classical first-order predicate calculus.
0thomblake11y
I think you're begging the question. I think you've given a definition of "classical logic" rather than "logic".
0Richard_Kennaway11y
I think that it seems that way only because classical logic has so definitively answered the question. The question is "how shall we reason?", and it was not obvious beforehand that the first-order predicate calculus was the answer. It took two thousand years to get there from the ancient Greeks' understanding.
0Rob Bensinger11y
My interpretation of this question is metaphysical: 'Is reality classical?' This is shorthand for: 'Is there any fact that is fundamentally, objectively, and in principle... ... inexpressible? ... vague? ... contradictory? ... indeterminate? (I.e., neither the case nor not-the-case.) ... etc. But this is a strange set of questions, and I suspect most professional philosophers and LessWrongers are instead answering a confused mixture of (mostly trivial) questions like 'which logic do I find most useful?'.
0RobinZ11y
Other: The objection to classical logic stated in pragmatist's summary needs to be addressed, but I lack sufficient knowledge of the field to determine whether the best resolution of these objections is likely to take the form of (a) a solution within classical logic or (b) the adoption of a non-classical logic.
-1[anonymous]11y
Other: probability distributions over world-histories. Otherwise classical.

A priori knowledge: yes or no?

[pollid:100]

5pragmatist11y
Yes: There are certain facts we can come to know for which our knowledge need not be based on sensory experience. No: The sort of justification that elevates belief to knowledge must always appeal to sensory experience.
0Rob Bensinger11y
This isn't quite right. As others have noted, 'sensory experience' is very ambiguous here. A better definition: Yes: There is at least one proposition one can be justified in believing merely by knowing the meaning of that proposition. I.e., no more experience is required than is necessary to understand the proposition in the first place. No: One cannot be justified in believing any proposition merely from knowing its meaning.
0pragmatist11y
The definitions you provide are traditionally associated with the analytic/synthetic distinction, not the a priori/a posteriori distinction. While many philosophers have held that all and only analytic propositions are true a priori, other philosophers have disagreed. Kant, for instance, regarded the truths of arithmetic and geometry as a priori true but not analytic, i.e. not true in virtue of the meanings of the terms involved. He thought our knowledge of these truths was based on the forms of our spatial and temporal intuition. There are also those who argue that certain moral truths are justified a priori (Kant was one of them). Again, I doubt that they would say that this is because their truth follows from the meanings of the terms involved. It's true that "sensory experience" is ambiguous, but that ambiguity has been at the heart of philosophical discussion about the a priori, so I think my definitions do capture the standard usage. I'd also note that "meaning" is similarly ambiguous.
2Rob Bensinger11y
I think both of our definitions are reasonably common, and that both are also somewhat misleading. I recommended mine partly because I've seen it from a lot of sources, e.g.: Penguin Dictionary of Philosophy, entry on a priori: "'It can be known a priori that p, if anyone whose experience is enough for him to know what "p" means, requires no further experience in order to know that p.'" Allen Wood, leading Kant scholar: "A proposition is known a priori when knowledge of it does not depend in any way on the specific contents of experience, when any experience that would suffice to enable us to entertain the proposition would also be sufficient to give us knowledge of its truth." So I think this 'quasi-analytic' definition is the more traditional one, though perhaps it doesn't fully capture modern usage. I also recommended it because I find the dispute over analyticity more philosophically interesting and deep than the dispute over what counts as sense-data. 'Is the experience of reasoning a sensory experience?' seems more obviously terminological than 'Is 2+2=4 true purely in virtue of its meaning?' You say that the ambiguity of the term 'senory experience' "has been at the heart of philosophical discussion about the a priori", but I think it's been mostly a blind-alley, and that the really substantive debates about the a priori have instead concerned the nature of justification in the absence of any experiential evidence, including memory, phenomenological introspection, telepathic insight, divine revelation... -- all of which fail to be conventionally 'sensory'. Or perhaps: Absent any experiential evidence aside from my experience with natural chains of reasoning and inference...? Here, I think, is where the heart of the dispute over the a priori lies: The question, not of whether our reasoning chains and intuitions are 'sensory,' but over whether they afford a categorically different kind of epistemic justification than does induction. The question is not 'W
3[anonymous]11y
A priori knowledge is knowledge from the operation of a mind. It is a sort of sensory experience.
1RobinZ11y
What, precisely, is the distinction between this and the "Knowledge: empiricism or rationalism?" question?
2Rob Bensinger11y
A good way of unpacking the distinction: A-priorism is a normative (evaluative, epistemological) thesis, whereas rationalism is a descriptive (factual, psychological) one. A-priorism says that we have warrant to believe some things without appealing to any evidence (more strictly: without appealing to any information beyond that which was required to understand the proposition in the first place). Rationalism says that we arrive at some of our understanding of reality without an essential causal dependence upon prior experience. (E.g., we have some extremely primitive proto-understanding of 'space' or 'causality' or 'quantity' that precedes our experiential acquaintance with the instances of those categories.) So an empiricist can assert a-priorism, if s/he thinks that in principle we could justify certain claims without any reference to experience, but also thinks that as a matter of fact our cognitive, epistemic, and conceptual grasp on everything, including our grasp on linguistic truths like 'all bachelors are bachelors,' stems entirely from sensory data. A-priorism doesn't entail rationalism. A rationalist must make the further assertion that some kinds of understanding are not only justifiable without appeal to empirical data, but are also obtainable without a causal basis in past empirical encounters. An empiricist might claim that our grasp of time, for example, developmentally arises via the sequence of external events imprinting itself upon the rudimentary sense-data-gathering faculties of the embryonic brain; whereas a rationalist would claim that we have some sort of grasp on time 'built in' by the evolved structure of our brain, requiring little if any 'pre-structured' sensory input to develop. Our experiential acquaintance with time is then mainly dependent on our innate makeup, rather than our innate makeup being mainly shaped by the temporality of our actual sense-data. (Notice that these are fuzzy distinctions; presumably actual brain developmen
0RobinZ11y
That makes sense - thanks!
1Risto_Saarelma11y
Hasn't this one been resolved by computer science?
1Jayson_Virissimo11y
I think Evo-Psych was first.

I think for future polls like this, mandate in the OP that all comments about questions be ROT13ed in order to avoid priming future respondents.

8DanArmak11y
That would be too cumbersome to use for the commenters.
2TheOtherDave11y
Agreed. I would be OK with having two posts, though, one of which is for discussion which people can therefore ignore until they've voted. That said, this whole site primes people for some of these questions.

Personal identity: physical view or psychological view?

[pollid:94]

Physical view: The maintenance of personal identity requires bodily continuity. So, for instance, one cannot preserve a person by downloading their psychological state into a computer.

Psychological view: The maintenance of personal identity requires continuity of psychological states. As long as there is a continuing stream of psychological states with the appropriate causal relations between them, the person persists.

7TheOtherDave11y
I answered "psychological", but I should perhaps note that I don't understand "continuing" to imply "uninterrupted". I have no problem with the idea of a personal identity that is shut down for a while before being booted back up (with its internal state saved), or one that is computed on a timesharing system.
2pragmatist11y
Yeah, I should have clarified. "Continuity" here does not mean temporal continuity; it means causal continuity. Future states are appropriately causally related to past states. So if I disintegrate right now, and simultaneously, by some bizarre chance, an atom-for-atom duplicate of me is produced by a thermal fluctuation on Mars, that duplicate would not be me, since the appropriate causal connections between my psychological state and his are lacking.
6TheOtherDave11y
Hm. My instinct is to say that I don't require causal continuity either... e.g., to say that if I appeared on Mars I would consider myself to still be the person who used to exist on Earth, despite the lack of causal connection. That said, I don't really take that instinct seriously, since I'm incapable of actually imagining this happening without positing a causal connection I just don't happen to be aware of. The alternative is... well, unimaginable. So, I dunno. Maybe I do require continuity in that sense, I'm just willing to posit it for any sufficiently complex system.
2faul_sname11y
So where does the information to build the copy of you on Mars come from? It's all fine and well to say "thermal noise" but if you allow for brains to be built from thermal noise with any sort of frequency, you end up with the bigger philosophical problem of Boltzmann brains. Unless you're proposing a mechanism by which the brain in question is your brain, in which case you've reintroduced causality.
2pragmatist11y
I agree that Boltzmann brains are a philosophical problem, but they're a problem precisely because our current best physical theories tell us that brains can fluctuate into existence. I don't think the right way to deal with the problem is to say, "Boltzmann brains are problematic, so let's just deny that they can exist."
1faul_sname11y
Yes, but our current best physical theories also mean that they probably fluctuate into existence considerably less often than they form under normal circumstances (human brains, at least). A mind is a complex thing, so the amount of information it takes to replicate a mind is probably far higher than the amount of information it takes to specify an environment likely to give rise to a mind. If you discard the causal process that gives rise to minds in practice and postulate thermal noise as the cause instead, you end up postulating Boltzmann brains as well. I didn't mean that Boltzmann brains are a particularly big philosophical problem, just that they become one when you try to do philosophy where you postulate very specific things occurring by "random chance".
2DanArmak11y
That's not how I understood the question. Now it turns out my vote is wrong. I should not have voted without understanding the question fully. But if I read the survey where the questions are taken from, I will probably also learn what is considered there to be the mainstream position, which will bias my answers.

Other: Leaning toward a causal view. In other words, your past self has to be the cause of your future self, but the specific atoms are irrelevant.

4[anonymous]11y
Holy crap! I'm identical with my kid!
4Eliezer Yudkowsky11y
Causal descent is a necessary but not sufficient condition, just like a QM-ignorant "physicalist" doesn't necessarily believe that if I grind you up and make a new person out of those "particular particles", it is the same person just in virtue of being made out the "same particles". Not that there's any such thing as the "same particles" in modern physics, just waves in a particle field, etc.
6[anonymous]11y
Right, but causal descent is common to the physical and psychological views. 'Physicalism' among philosophers generally doesn't refer to some kind of 'same atoms' view. That's an incoherent view long before we bring in considerations of quantum physics, and the 'same particles' issue. Mostly that kind of physicalism is restricted to people who are wrong on the internet. Physicalism among (most) philosophers who hold that view is the claim that your identity is tied to a particular animal (or whatever hardware) that has physical persistance conditions (like the processes which keep it alive, etc.). If you create an atom-for-atom duplicate of that animal, and then kill one of the two of them, you haven't therefore killed both of them. They're not identical in that sense, and that's the sense of 'identity' that physicalists are calling personal identity. So nothing about quantum physics, so far as I can see, makes a difference to this question.
0Thomas11y
Holly crap, identical withe everybody ever lived. Except those of course, who were not self aware. If such exist.
3[anonymous]11y
Based on pragmatist's interpretation, this sounds like the physical view.
2pragmatist11y
It sounds like the psychological view to me, although I guess that depends on what Eliezer means by "self".
2MugaSofer11y
This confuses me. I'm a bunch of LessWrong posts? I voted Accept: psychological view.
1[anonymous]11y
Voted other for essentially this reason. Still very confused about this question.
9Kaj_Sotala11y
I answered "lean toward psychological view", as I'm about evenly split between the psychological view and just considering the whole concept of personal identity incoherent/a wrong question in the first place.
0[anonymous]11y
Same. I think I might possibly be describable as more of an anticriterian than anything else.
3Matt_Simpson11y
Other: I suspect the answer to this question depends on the particular question you're asking. Often, I think, this is a values question - e.g. in what form do I want to continue existing?
3asparisi11y
Other: I think that personal identity is in a certain sort of brain process, rather than a static "brain state." (so, no "static minds") The physical view implies that personal identity can be reduced to certain brain states, and I reject that: a valid sort of active processing is important.
2Mutasir11y
Physical view/other (?). I consider personal identity to be based only on the memory of past self, not actual brain process or existence of memory independent conscious mind. It may be very well that the experience of personal continuity is only an illusion as there is no actual continuity at all, only the recollection of previous events (including the attempts of introspection). An exact copy of me wouldn't be any different than myself.
2Caerbannog11y
Other: For everyone else that I observe, an exact atom-for-atom duplicate is the same person as the original. If a copy of me were made, my 'mind' would reside in the original. I accept that my duplicate would claim to be the original, of course.
4TheOtherDave11y
I'm not sure what you mean by "the original" here. Suppose the atom-for-atom duplicate were constructed (for sound technical reasons) inside a duplication chamber, and it came to awareness inside that chamber. Would it claim that it had somehow had been swapped into the chamber and the duplicate swapped out, without it noticing? Or would it acknowledge that it had been constructed in the duplication chamber, but claim to be the original nonetheless?
2Caerbannog11y
Whether the duplicate claimed to be the original or not depends on the individual, I suppose. If I lived in a world that contained such duplication chambers, and found myself waking up in one, I would not know whether I was "the copy" or not without some outside evidence. I'd be inclined to accept that either I was a copy, or someone was playing a trick on me to make me think so. I understand that the duplicate would have the same memories and personality as me, but would not have my subjective sense of experience.
0TheOtherDave11y
OK, now I'm confused. You said in this case you'd be inclined to accept that either you were a copy, or someone was playing a trick on me to make you think so. Which makes sense. Would your duplicate be equally inclined to do the same thing in the same case? If so... then why would your duplicate claim to be the original? If not... what accounts for the difference?
2Caerbannog11y
Yes, my duplicate would think the same way as me. In a world that has duplicators, my duplicate would not claim to be original without evidence one way or the other. In our real world, if a copy of me were made using "magic", both versions would believe themselves to be the original (at least at first). I had this kind of very specific scenario in mind when I said both would claim to be original, but did not explain this in the earlier comment (inferential distance and all that).
2[anonymous]11y
That's not 'other', that's the psychological view (assuming you would still say they're the same person if one was duplicated minus a left pinky).
-1Caerbannog11y
I don't know: If someone I knew had their physical body destroyed but they were uploaded with complete accuracy, I would consider them to be the same person (consistent with psychological view). I would not opt for that procedure for myself, though, because I don't accept that my upload would really be me (more like physical view). I'm open to evidence and argument on this, though.
3MugaSofer11y
This is consistent how?
3TheOtherDave11y
Well, I'm not obligated to use the same standards for myself as other people.
1Caerbannog11y
I can observe myself in a way I that can't others. From my vantage point, a copy or upload of someone else behaves the same as the 'original'. From that same vantage point, a newly created copy of myself is clearly 'outside' my mind and therefore observationally different.
0MugaSofer11y
But surely the copy is as much the same person as the "you" of five minutes ago as the original?
-1Caerbannog11y
To you and everyone else, but not to me.
1MugaSofer11y
They may not be you(now), but if you count yourself as the same person as you(earlier), then they have to be the same person as you(earlier) as well. I think.
1Caerbannog11y
A newly created copy or electronic upload of me (call him 'Copy B') would have all my behavioral attributes and memories. He could be called $myName by anyone else observing either of us (we could be indistinguishable to a third observer). However, to me (the guy writing this response, call me 'Copy A'), there would be an obvious observable difference between Copy A and Copy B. I see the world from Copy A's point of view, with his eyes and ears and I would observe Copy B from the outside as I would any other person, without knowing what is going on in his mind or experiencing the world from his point of view. Yes, Copy B might say the same about Copy A, but it's my fear that Copy A would never find himself genuinely waking up inside a copying chamber or as an upload. If that's true, uploading myself would be the death of my subjective point of view. I get where you're coming from. I don't necessarily have an epiphenomenal view of the mind, but I also believe that the concept of qualia is not well understood by anyone. I do not understand why I'm me and not someone else, and neither does our current knowledge on the subject. Based on this I'm agnostic on whether mind uploading in the style we're discussing would really preserve me and my stream of qualia, or kill me and create another person with a new stream of qualia. Without any evidence that it would preserve me, I would not accept going through such a process. There are possible scenarios in which the copying process could preserve what I consider to be me: For example, if there is only one observer at all, who experiences all qualia streams throughout the world (that possibility scares me, honestly). Another possibility might be that copying me would simply double my measure in the world, and what I consider my qualia stream would have twice as many experiences after the copying process. These are just speculation, though. This has definitely been an interesting discussion for me. Examining my thoughts on
0Epiphany11y
I had the same reaction, but the majority of others I've talked to disagree with me, so it's nice to see someone who thinks the same way. Here are my arguments with TheOtherDave (Ironic, I know!): Teleporter Malfunction Scenario
0MugaSofer11y
No, I understand that, I'm saying that, while Copy B is not the same person as Copy A, he IS the same person as Copy A was before being copied, at least as much as Copy A is. What would you do if you discovered you were Copy B in such an experiment? Because presumably he would do the same thing.
1Caerbannog11y
I don't contest your first paragraph. Regarding your question: I don't know. Probably update my understanding of this subject.
1magfrump11y
Other: Psychological states include some bodily states like sensory input, and possibly as much as "social contact." There is no firm boundary between physical and psychological. Uploads are possible, but will require emulation of more than just a brain. An atom-by-atom instantiation of the same mind (including enough of the body and environment) will be the same person.
0Nisan11y
Psychological view: This question is standing in for a particular question of values. Otherwise it's meaningless.
0DanArmak11y
What is the psychological view, if it's non-physical(ist)?
1benelliott11y
I believe, the distinction is whether 'you' are the physical atoms that make up your brain, or just the pattern those atoms form. So you were instantaneously replaced by an identical copy, would you still be the same person? Has implications for things like uploading.

Metaphilosophy: naturalism or non-naturalism?

[pollid:91]

6DanArmak11y
I was going to post a snarky comment to the effect that if you discard outright religious views and cognition motivated by them, there doesn't seem to be much left to non-naturalism. But the Stanford Encyclopedia of Philosophy says it better: So non-naturalism looks like mostly a combination of religion and arguments over the meaning of the word "natural". As in, if we found evidence that spirits of the dead affected physical events, that would promote them to the status of natural physical phenomena. So of course everything that exists is "natural" - according to some definitions of the word. What is there to non-naturalism that is worth the time of seriously investigating it?
0[anonymous]11y
How would you investigate it?
0Richard_Kennaway11y
One could say the same thing of a lot of philosophy.
1Kindly11y
What is a naturalism?
0Jayson_Virissimo11y
Other: Naturalism is the appropriate stance to take with respect to our own world, but our own world is (probably) not all there is. If the simulator were to intervene, then these interventions could reasonably be described as being supernatural. Does this view have a name already? If not, I nominate relativistic naturalism.

Knowledge: empiricism or rationalism?

[pollid:90]

Empiricism: Our only source of novel information about the world is sensory experience.

Rationalism: There is some information about the world that we can arrive at by rational cogitation, without having to rely on sensory experience.

-2Eugine_Nier11y
Isn't the standard formulation of "Rationalism" that all information can arrive at by rational cogitation, without having to rely on sensory experience? Yes, I know no one sane believes that formulation these days.
1pragmatist11y
That's traditional rationalism of the Cartesian variety, but it's not what a contemporary philosopher would mean if she called herself a rationalist. I can guarantee that the vast majority of the respondents to the PhilPapers survey who answered "rationalism" to this question do not believe that one can arrive at all information without having to rely on sensory experience.
6Caerbannog11y
Other: Agree that it's a false dichotomy.
6FiftyTwo11y
How? If there is >1 fact "we can arrive at by rational cogitation, without having to rely on sensory experience." Then rationalism, if not then Empiricism.
1thomblake11y
Most of the "false dichotomy" claims here are from folks claiming it's an "unheard tree in the woods" problem - if you use one definition, you get one answer, and if you use another definition, then it's another answer, so if we just tabooed the relevant words then the dispute would go away.
1[anonymous]11y
Agree on other. Rational cogitation is a kind of sensory experience.
6jimrandomh11y
I believe this to be a false dichotomy; both empirical and rational processes generate knowledge, although particular kinds of knowledge (such as the colors of objects or the truth of theorems) may be restricted to arising from one or the other.

This position is basically rationalism. Contemporary rationalists don't deny the possibility of empirical knowledge. That would be a fairly absurd position to hold in the present. They say that there are also non-empirical sources of knowledge. Empiricists deny the existence of non-empirical sources of knowledge.

8TheOtherDave11y
Suppose I flip a coin 999 times and it comes up heads. I then flip it a hundredth time but don't look at it. I would be comfortable saying I know without looking that it came up heads. (Sure, there's a chance I'm wrong. There's a chance I'm wrong if I look, too. If "knowledge" denotes the state of absolute certainty, we don't ever know anything. It makes more sense to interpret "knowledge" as denoting greater-than-threshold confidence.) Would a contemporary empiricist say that I don't know that, because I didn't see it? That I know this, but it's not novel information? That it's novel information, but I obtain it through sensory experience? (E.g., observing the previous 999 flips) Other?
5pragmatist11y
I think the contemporary empiricist would say that all the information you have about the thousandth flip comes from your past sensory experience -- your experience of the previous 999 flips plus other relevant experience (such as, say, experiences that form the basis for your beliefs about the base rate of unfair coins). The extent to which your belief about the thousandth flip justifiably differs from maximum entropy (or zero information) is entirely attributable to your prior experiences.
3TheOtherDave11y
OK, so an contemporary empiricist doesn't deny the possibility of inference. Good. Does a contemporary empiricist deny the possibility of inference engines being constructed in ways that bias them towards certain conclusions? E.g., that two people might be born with their brains wired such that, given the same sensory experiences, one of them infers A and the other infers B? (In both cases, presumably, the information about A or B comes from past sensory experience, it's just that the process for getting one from the other differs.) If not, then I no longer have a crisp sense of what contemporary empiricists and rationalists actually disagree on.
4pragmatist11y
Like I said in another comment, I identify as a rationalist because empiricism, construed literally, does not allow for informative priors, which makes learning impossible. I'm pretty sure, though, that if you brought this up to a philosopher who identifies as an empiricist, the response would be "Well of course that's not what I mean by empiricism. Informative priors are fine." But then, like you, I'm not so sure how to interpret the rationalism/empiricism distinction.
2Rob Bensinger11y
Given your definitions of 'rationalism' and 'empiricism,' an empiricist would need to assert that informative priors, if they exist, either are not "novel information about the world" or are novel information that we derive from experience. We aren't perfect Bayesian reasoners, and you haven't defined 'information,' so this doesn't seem perfectly open-and-cut to me. One approach an empiricist could take would be to deny that our primordial priors (i.e., our earliest expectations), in themselves, constitute information about the world; perhaps we can use them as a handy framework for genuinely informative research, but the framework itself is not knowledge, Another approach would be to deny that we have expectations before possessing any sense-perception; perhaps neurological development relies extensively upon sensory input from our environments before anything as cognitively complex as 'expectation' or 'belief' enters the picture. Or one could adopt a mixed strategy.
2shminux11y
What are those, besides instincts? Any examples?
5pragmatist11y
I call myself a rationalist because learning is impossible with maximum entropy priors, so if we can learn about the world through experience, we must start out with informative priors, which means we have some information about the world that is not attributable to experience. However, I suspect that this kind of position would not be recognized as rationalism by many philosophers. A more traditional rationalist claim is that reason can provide us with novel information about the world. As an example, consider a Platonist who believes that the integers actually have some kind of independent, objective existence, and aren't just the elements of a useful formal system constructed by humans. In that case, someone who proved Fermat's Last Theorem would have discovered a fact about certain objects in the world, but not through sensory experience.
2TimS11y
Isn't this what EY argues for at the end of QM sequence? He seems to think there are ways of knowing things when empirical evidence is insufficient to resolve the dispute.
-1shminux11y
Right, that's where he loses me every time. We disagree on what "knowing" means.
2TimS11y
Whereas I say that EY's position in the QM sequence would be right - if rationalism were more correct than empiricism. Of course, I think your position on "knowing" is much too practical :) The fact that resolving physical realism vs. anti-realism doesn't pay rent at the engineer's bench does not mean it doesn't matter to Science. Whereas you are a hardcore instrumentalist. I'll grant you that rationalism vs. empiricism is not a well-formed question if one is an instrumentalist.
0shminux11y
Well, we agree on something. Just to clarify, my instrumentalist approach comes from the frustration of not being able to argue "which model is correct?" without tying correctness to testability. I was a naive realist a year or so ago, before I started reading this forum regularly.
2TimS11y
Sure - falsifiability is the key issue. I think that the physical realism sides would make different predictions about the process of scientific progress. So we compare those predictions to the actual data from the history of science. I happen to think Kuhn and Feyerabend make the better argument about how to interpret the history, so I'm an anti-realist. If one thinks Kuhn and Feyerabend made a mess of the history, realism is a much more appealing position. I almost think pragmatist didn't go far enough in his explanation of the difference.
0shminux11y
Is there a way to unambiguously test this assertion?
2DanArmak11y
Should we be discussing the questions before others have had a chance to vote without being swayed?
4jimrandomh11y
I meant this as an explanation of my 'Other' vote, but yes we should discuss, because postponing discussion is not a realistic option - comments will go unwritten, rather than being delayed. Spoiler tags would be helpful, but I don't think we have them.
2DanArmak11y
You're right.
2Richard_Kennaway11y
Yes, if it elucidates what the questions mean.
5Eliezer Yudkowsky11y
Other: What the hell does Solomonoff Induction count as?
8gwern11y
I'd call it empirical, without a doubt: it's as 'unrational' as possible, since it admits every possible computable sequence and selection of which one is determined by observations. If that isn't empirical...
0Manfred11y
But is "as un'rational' as possible" a'rational'?
2MugaSofer11y
I'd say rational, since it's a priori.
5Manfred11y
Rationalism: doesn't work for arbitrary minds, but works for us, as we aren't arbitrary minds (yay evolution).
3novalis11y
Other: I would tend to regard our reason as a sense.
4DanArmak11y
So that's the mysterious common sense people talk about!
2FiftyTwo11y
Surely you should just substitute in "our other senses"?
0novalis11y
I think of rationalism (in this sense) as thinking of reason as more distinct from (say) vision than I think of it as.
-1bramflakes11y
Other for basically the same reason as this, though I never thought of it in those words.
-2[anonymous]11y
Then you're an empiricist.
3pragmatist11y
I would say it's more like novalis thinks there is no substantive distinction between empiricism and rationalism.
0[anonymous]11y
He definitely thinks there's a substantive difference: if reason is a sense, and all our knowledge comes from the senses (including reason) then all our knowledge is a posteriori. Rejecting the mechanism of a priori knowledge acquisition is rejecting rationalism (regardless of how the word 'rational' mutates in the mean time).
2lucidian11y
Other: rationalism with a caveat of embodied cognition.
2drnickbone11y
Went for "Other". Bayesian updating appears to be a mixture of "rationalism" and "empiricism".
1Manfred11y
To quote Esar ( :D )
0drnickbone11y
I wasn't sure whether the calculations made between observations (the updating of probabilities) should count as "new information about the world" or not. From a strictly information theoretic point of view they don't (since the calculations are entailed by the observations so far, there's no reduction in Shannon entropy after making them). From a psychological point of view they do, since we learn as much - or more - from the updates as we do from the observations themselves.
0RobinZ11y
Other: Although human minds can discover facts about the world through non-sensory processes, their ability to do so only exists through the action of evolution by natural selection - and natural selection is a basically-empirical process.
0asparisi11y
Other: Empiricists about knowledge like to claim that knowledge is due to sensory experience, while rationalists claim it to be intrinsic a priori. I see knowledge as an active process of updating: it's heavily laden with sensory experiences but you need some starting state, and I think that starting state is an implicit part of your knowledge. The two aren't separable.
0[anonymous]11y
Rationalists claim that some but not all knowledge is a priori. So I think your position might be rationalism.
3asparisi11y
That depends on the Rationalist (Spinoza arguably denies this in his idealism, and one could argue that Plato is a rationalist who believes that all knowledge is a priori.) but the point here is that I think that knowledge always has an empirical part and a rational part. In other words, I reject the a priori/a posteriori demarcation.
-2A1987dM11y
Other: both.
4[anonymous]11y
That's rationalism. 'Both' would be a contradiction.
0A1987dM11y
I mean: I think that all knowledge about the world requires both empirical evidence and reason, and no knowledge can come from either alone.

Free will: incompatibilism or compatibilism?

[pollid:82]

Incompatibilism: One cannot have free will in a deterministic universe.

Compatibilism: One can have free will in a deterministic universe.

0Rob Bensinger11y
So I'm an incompatibilist: One cannot have free will. Hence, one cannot have free will in a deterministic universe.

Other: "Free will" is a confused term, so the question is unanswerable. If one takes the mechanism that results in the confusion about free will and labels that "free will", then of course compatibilism holds.

5[anonymous]11y
I voted 'accept compatibilism' but essentially agree with this statement.
0Rob Bensinger11y
In no other field of discourse that comes to mind do we generally take a non-existent thing P and re-label the psychological cause of belief in that thing 'P' merely so that we get to keep using the word. What would be the precedent? If P is a confused term, then asserting 'P exists' is either false or meaningless, not 'trivially true because we can redefine it in some principled way'.
3Oscar_Cunningham11y
"Centrifugal force"
0Rob Bensinger11y
Centrifugal force isn't a psychological cause. It's not as though we learned that centrifugal force is a hallucination caused by iron and B12 deficiency and redefined 'centrifugal force' to mean 'a deficiency of iron and B12.' Redefining 'free will' to mean 'the sensation of feeling as though one's actions are in no way determined by causes outside oneself' (or whatever it is that we're supposed to be salvaging here) strikes me as more similar to redefining 'sin' to mean 'the sensation of feeling guilty' or perhaps 'the sensation of feeling that one has transgressed against a supernatural order.' Or redefining 'karma' to mean 'the intuition that we live in a just universe.' Or redefining 'soul' to mean 'the brain's tendency to treat itself as though it were immaterial.' Where do we take metaphysical or religious ideas and salvage them by re-assigning them cognitive states? Maybe I'm missing some obvious pool of cases of this sort.
0RobertLumley11y
Upvoted in agreement. (I responded other as well.)
9bramflakes11y
Other, purely because in reading pragmatist's definitions I also accidentally read two other comments which confused me. My position is that we do not have contracausal free will, yet our decisions feel like that from the inside, and that indeed the question itself is usually a disguised query for whether we can ever be morally responsible for things, which I believe is a separate question.
2asparisi11y
I said "Lean toward: incompatibalism" because I assume you are talking about Libertarian or Contra-Causal Free Will when you talk about Free Will, but are also including less-strong theses about Free Will. If you are not including contra-causal free will, change it to "Accept Compatibalism" if you are ONLY talking about contra-causal free will, change it to "Accept Incompatibalism."
1Spinning_Sandwich11y
I like the idea of pulling some language from logic and saying we have "bound will," not "free will." This may well be compatibilism as intended by its defenders, but that isn't the impression I've ever had from their papers. I would [very] roughly describe bound will with the following two claims: My will is free from Susie's will. Neither Susie's will nor my will is free from physical causes. Notice that such a term doesn't care whether the universe is strictly deterministic or merely stochastic.
0DanArmak11y
Other: it depends entirely on what one defines "free will" to mean, and all disagreements on the question are due to different definitions. Nobody is disputing actual facts. (That is, nobody who accepts a deterministic or random universe, as specified in pragmatist's definitions.)

Retracted. Sorry. Didn't read post properly.

[This comment is no longer endorsed by its author]Reply